Patho Physio 51 to 100q

August 8, 2018 | Author: Raquel Bencosme | Category: Heart Valve, Adrenal Gland, Heart, Cortisol, Organ (Anatomy)
Share Embed Donate


Short Description

uworld...

Description

USMLE WORLD STEP 1

PATHOPHYSIOLOGY

Q NO 51: A 6-year-old Caucasian girl is brought to your office by her mother. The mother says that her daughter is “womanly” including being exceptionally tall for her age. Physical examination reveals Tanner Stage 3 breast development and coarse pubic hair. The child’s height is in the 96th percentile. You explain to her mother that, although this girl is yew tall now, she will ultimately be shorter than average if she does not get treatment. Which of the following is the physiologic explanation for the information you provided? A. Estrogen effect on the long bone diaphysis B. Estrogen effect on the long bone epiphysis C. Estrogen effect on the long bone epiphysial plate D. Somatomedin C effect on the long bone diaphysis E. Somatomedin C effect on the long bone epiphysial plate Explanation: The child described in the vignette is six years old and has already undergone pubertal changes, as indicated by her Tanner staging. (Also called the sexual maturity rating, Tanner staging is a systematic, universalized method that uses female breast and pubic hair development to categorize development.) Precocious puberty in Caucasian females is defined as the development of secondary sexual characters at an age less than seven years. Young bone has several layers; their order from middle to end is diaphysis, metaphysis, epiphyseal cartilage, and epiphysis. The epiphyseal cartilage, also called the “growth plate, is responsible for linear growth. Linear growth stops when the epiphyseal growth plate closes, i.e. when the epiphysis fuses with the metaphysis. Sex steroids initially increase linear growth, but they also encourage closure of epiphyseal growth plates. Once the growth plates have closed linear growth is irreversibly stopped. In normal puberty there is a rapid increase in growth. At the end of this pubertal growth spurt, almost 90% of the final height is achieved. Although this child is currently taller than her peers the excess steroids will soon cause epiphyseal growth plate closure: hence, if this patient is not treated, she will ultimately not meet her full growth potential. (Choices A and B) in sites other than the epiphyseal growth plate the effect of estrogen on bone is anabolic because estrogen increases osteoblastic bone deposition and decreases osteoclastic bone resorption. Low estrogenic states, such as menopause, are associated with rapid bone loss. (Choices D and E)The secretion of somatomedin C, more commonly known as insulin-like growth factor-i (IGF-i), is directly increased when growth hormone is secreted from the pituitary gland. IGF-i causes the differentiation and proliferation of chondrocytes in the epiphyseal growth plate, causing an increase in linear growth. However, unlike estrogen, IGF-1 does not accelerate epiphyseal closure. Educational Objective: Sex hormones promote both growth and epiphysial plate closure: hence, precocious puberty may result in a shorter stature, despite an initial growth spurt. Gigantism is caused by excessive pituitary production of growth hormone; these patients achieve enormous heights because, unlike excessive sex steroids, excessive IGF-1 does not lead to premature closure of the epiphysis.

249

USMLE WORLD STEP 1

PATHOPHYSIOLOGY

Q NO 52: Animal experiments have shown that chronic, chemical, intravascular inju results in intimal thickening and collagen deposition. Which of the following cells are most important in this intimal response? A. Interstitial fibroblasts B. Endothelial cells C. Smooth muscle cells D. Macrophages E. Pericytes Explanation: The vascular reaction to intimal injury, including endothelial injury, involves migration of medial smooth muscle cells (SMC) across the internal elastic lamina and into the intima, followed by SMC proliferation and collagen synthesis to produce a neointima. The result is often a reactive intimal hyperplasia. Injured endothelial cells may release growth factors, such as platelet-derived growth factor (PDOF) that promotes SMC migration and proliferation. They may also express surface vascular cell adhesion molecules, such as VCAM-1 which allow adherence of monocytes and lymphocytes and their diapedesis into the intima. These white blood cells can then release further cytokines and growth factors to promote SMC migration into and proliferation in the intima, encouraging subsequent fibrogenesis (elaboration of collagen and extracellular matrix). Finally, when intimal injury is severe enough to denude the endothelium, there is platelet adhesion to subendothelial collagen and platelet release of PDGF. (Choice A) Fibroblasts are infrequently found in the tunica intima of blood vessels. In response to injury, medial SMC migrate into the intima, and are responsible for intimal thickening and collagen deposition. (Choice B) Injured endothelial cells may secrete factors that promote medial SMC migration and proliferation, including PDGFI FOE, and endothelin-1. However, the SMC are directly responsible for intimal thickening after intimal injury as a result of SMC proliferation and synthesis of new collagen, elastin, and proteoglycans. (Choice D) SMC derived from the media are responsible for intimal thickening. After injury, these cells migrate, proliferate, and synthesize new connective tissue. However, tissue macrophages in the intima can contribute to this process as well. Blood monocytes can be recruited to the intima beneath sites of endothelial injury to form tissue macrophages. Educational Objective: The vascular reaction to endothelial and intimal injury is intimal hyperplasia and fibrosis, mediated predominantly by reactive SMC that migrate from the media to the intima.

250

USMLE WORLD STEP 1

PATHOPHYSIOLOGY

Q NO 53: In certain adults, the myocytes of the cardiac ventricles express mRNA for natriuretic peptides typically synthesized by the atria. This finding is associated with which of the following conditions? A. Hyperplasia B. Hypertrophy C. Malnutrition atrophy D. Sublethal ischemic injury E. Reperfusion injury F. Normal aging Explanation: Pathologic ventricular hypertrophy is accompanied not only by morphologic changes, but by changes in gene transcription patterns as well. When there is prolonged hemodynamic overload, there may be abnormal up-regulation or re-expression of fetal proteins from both atrial and ventricular myocytes. These alterations in protein synthesis may be adaptations to improve contractile efficiency and to decrease ventricular wall stress. In the normal adult heart, A-type natriuretic peptide (ANP) is secreted by the atria, and in some cases the ventricles in response to volume overload. B-type natriuretic peptide (BNP) is predominantly secreted by the ventricles. Volume overload increases the release of both ANP and BNP, facilitating natriuresis and diuresis. BNP levels can be used for the diagnosis of congestive heart failure (both systolic and diastolic). (Choice A) The cardiac myocyte is generally considered a terminally differentiated cell that can no longer divide. Increased mechanical loads (pressure and/or volume overload) can increase the sarcomere content and volume of individual ventricular myocytes (hypertrophy), but do not cause hyperplasia. (Choice C) Atrophy from malnutrition causes a reduction in the structural components of the cell. The myocytes of atrophic muscle contain fewer myofilaments and mitochondria, and less endoplasmic reticulum. (Choice D) Reversible ischemic injury to cardiac myocytes results in intracellular ATP depletion glycogen loss, lactate accumulation relaxation of myofibrosis and mitochondrial swelling. There are no associated increases in ANP or BNP. (Choice E) Reperfusion injury is thought to result from generation of oxygen-free radicals and/or local complement activation. ANP and BNP are not elevated in this condition. (Choice F) Normal aging of ventricular myocytes results in atrophy and the progressive accumulation of cytoplasmic granules containing brownish Lipofuscin pigment. Educational Objective: 1. Both ventricular hypertrophy and volume overload cause release of both ANP and BNP from the ventricular myocytes to facilitate natriuresis and diuresis. 2. Reperfusion injury is thought to result from generation of oxygen-free radicals.

251

USMLE WORLD STEP 1

PATHOPHYSIOLOGY

Q NO 54: A 28-year-old Caucasian female comes to your office complaining of constant thirst. She also urinates a lot, and her mouth is very often dry. Her blood glucose level is 80 mg/dL, and her plasma sodium level is 132 mEq/L. You proceed with the standard water deprivation test in this patient. The urine osmolality results of the test are presented below. 1 hour 2 hour 3 hour 4 hour 5 hour 6 hour 7 hour 8 hour 130 160 170 285 310 510 510 520 5 units of vasopressin administered Which of the following is the best treatment for this patient? Desmopressin B. Hydrochlorothiazide C. Indomethacin D. Low salt, low protein diet E. Water restriction

A.

Explanation: Three important causes of polyuria and polydipsia are diabetes mellitus, diabetes insipidus (Dl), and primary (psychogenic) polydipsia. Diabetes mellitus can quickly be ruled out in this patient because her blood glucose is normal. Next, one must decide between Dl and psychogenic polydipsia. Dl is an inability to concentrate the urine because of either a deficient amount of vasopressin (antidiuretic hormone [ADH], or an abnormal response to ADH. In a water deprivation study, patients with Dl will not have a significant change in urine osmolality. Primary polydipsia, called psychogenic polydipsia in older literature, is simply excessive (pathologic) water drinking. It is a psychological disorder found more commonly in women and children and cannot be diagnosed if there is a medical reason the patient feels thirsty—for instance many medications give patients “dry mouth h In real practice primary polydipsia is a diagnosis of exclusion and should be diagnosed cautiously. For test purposes however one can identify primary polydipsia easily: there will be a steady, reliable and prompt increase in urine osmolality during water deprivation studies and there will be a paltry response to vasopressin administration. Both results were true for this particular patient. Additionally patients with primary polydipsia will have low serum sodium and low serum osmolality. Restriction of water intake normalizes urine output in patients with primary polydipsia. (Choice A) The pathology of central Dl is lack of adequate ADH. Desmopressin is a synthetic form of ADH and the treatment of choice for central Dl. (Choices B and C) Nephrogenic Dl is caused by a lack of response to ADH. Hydrochlorothiazide causes increased water absorption from the proximal tubule (remember ADH acts in the collecting tubule) and is used to treat nephrogenic Dl. Alternatively indomethacin is sometimes used; it increases water absorption by decreasing prostaglandin synthesis (prostaglandins inhibit ADH). (Choice D) Low salt low protein diets are typically used in patients with renal failure. Educational Objective: During a water deprivation test most patients with primary polydipsia will demonstrate a significant increase in urine osmolality. Additionally patients with primary polydipsia will have low serum sodium and low serum osmolality. Restriction of water intake normalizes urine output in patients with primary polydipsia.

252

USMLE WORLD STEP 1

PATHOPHYSIOLOGY

Q NO 55: A 61-year-old male presents to your office with a history of progressive dyspnea. He can not tolerate moderate exertion and sleeps in a half-sitting position due to orthopnea. He has also noticed some swelling of his ankles. He does not smoke or consume alcohol. His past medical history is significant for hypercholesterolemia and recurrent chest pain. Which of the following do you expect to be increased in this patient? A. Cardiac output B. Renal perfusion C. Arteriolar resistance D. Mixed venous oxygen content E. Lung compliance Explanation: The symptoms described in this clinical scenario are strongly suggestive of congestive heart failure (CHF). Left-sided CHF frequently presents with dyspnea on exertion, orthopnea (dyspnea while lying flat) and paroxysmal nocturnal dyspnea (waking from sleep gasping for air). Symptoms of right-sided CHF include lower extremity edema and hepatomegaly. Most commonly combined left- and rightventricular failure is observed owing to the fact that the most common cause of right heart failure is left heart failure. CHE occurs due to a progressive decrease in myocardial contractility (systolic dysfunction) or due to inability of heart to relax sufficiently to accommodate incoming blood in diastole (diastolic dysfunction). CHE of both origins ultimately leads to decreased cardiac output and inadequate oxygen delivery to tissues. Decreased perfusion of peripheral tissues induces a number of neuroendocrine compensatory mechanisms, such as: 1. Renin-angiotensin-aldosterone mechanism: Decreased renal perfusion leads to diminished stretch of glomerular afferent arterioles and increased synthesis of renin. Increased renin is converted to angiotensin II, which is a potent vasoconstrictor. By causing vasoconstriction, angiotensin II increases arterial resistance and after load (Choice C). The failing heart is unable to pump blood to the tissues against the increased afterload induced by angiotensin II, so tissue perfusion decreases and more renin is secreted by the kidney creating a vicious circle. Angiotensin II also stimulates the secretion of aldosterone which functions to increase the circulating blood volume (preload) and further exacerbate CHR 2. Increased sympathetic output is stimulated by baroreceptors that sense decreased perfusion. Epinephrine and norepinephrine increase heart rate and contractility but these neurotransmitters also increase peripheral arterial resistance thereby increasing afterload. (Choice A) Cardiac output is decreased in both systolic and diastolic heart failure. (Choice B) Decreased cardiac output leads to diminished perfusion of the viscera, including the kidneys. Decreased renal perfusion activates the renin-angiotensinaldosterone mechanism. (Choice D) As less arterial blood is delivered to peripheral tissues the contact of tissues with venous blood is increased. Uptake of oxygen from venous blood is higher, and mixed venous oxygen content is decreased. (Choice E) Increased pressure in pulmonary circulation results in transudation of fluid into the lung interstitium and air spaces. This decreases lung compliance and oxygen diffusion.

253

USMLE WORLD STEP 1

PATHOPHYSIOLOGY

Educational Objective: Decreased cardiac output triggers a number of compensatory mechanisms. Reninangiotensin-aldosterone activation and increased sympathetic output raise arterial resistance (afterload) and exacerbate heart failure by making it more difficult for the failing heart to pump blood to the tissues.

254

USMLE WORLD STEP 1

PATHOPHYSIOLOGY

Q NO 56: A 46-year-old Caucasian female is hospitalized for recurrent renal colic. Her past medical history is significant for peptic ulcer disease for which she takes famotidine daily. Her laboratory values are: Serum sodium 140 mEq/L Serum potassium 4.0 mEq/L Chloride 103 mEq/L Serum creatinine 0.8 mg/dL Serum calcium 12.0 mg/dL Serum phosphorus 2.4 mg/dL 24-hour urinary calcium excretion 350 mg (N 100-300 mg) Which of the following changes in bone structure is most typical for this patient’s condition? A. Osteoid matrix accumulation around trabeculae B. Trabecular thinning with fewer interconnections C. Subperiosteal resorption with cystic degeneration D. Lamellar bone structure resembling a mosaic E. Spongiosa filling medullary canals with no mature trabeculae Explanation: The clinical features described here are most consistent with primary hyperparathyroidism. Primary hyperparathyroidism is caused by a parathyroid adenoma in 80-85% of patients and by parathyroid hyperplasia in the remaining 1O15%. (Parathyroid cancer is a very uncommon cause of primary hyperparathyroidism.) The excess serum calcium found in hyperparathyroidism occurs by three mechanisms: (1) an increase in the renal absorption of calcium, (2) an increase in the gastrointestinal absorption of calcium indirectly, by 1, 25dihydroxjvitamin D formation (parathyroid hormone increases the formation of 1, 25 dihydroxyvitamin D in the kidneys), and (3) an increase in bone resorption by osteoclast activation. The serum phosphorus in patients with hyperparathyroidism is usually low because parathyroid hormone decreases phosphate resorption in the proximal renal tubule. For unclear reasons cortical or compact bone, in the appendicular skeleton (the pectoral girdle, the pelvic girdle, and the limbs) is more commonly involved in hyperparathyroidism. Subperiosteal thinning is a characteristic feature of hyperparathyroidism. Radiologically this thinning appears as subperiosteal erosions in the medial side of the second and third phalanxes of the hand, and there is a granular “salt-and-pepper” appearance of the skull. The cardinal clinical manifestations in patients with primary hyperparathyroidism, are bone loss, renal stones, gastrointestinal upset (ulcers), and psychiatric disorders (“bones, stones, groans and psychic moans”). These symptoms are described best by older textbooks because most cases today are detected before any bone or kidney symptoms occur. The present practice of routinely measuring calcium in the chemistry profile allows patients to be diagnosed well before the onset of clinical symptoms. (Choice A) Histologically, vitamin D deficiency is characterized by excessive un mineralized osteoid and widened osteoid seams. Patients with vitamin D deficiency have low bone mineral densities and are at risk for fractures. (Choice B) Trabecular thinning with fewer interconnections is characteristic of osteoporosis. In osteoporosis, the total bone mass is decreased, and normal bone architecture is disrupted.

255

USMLE WORLD STEP 1

PATHOPHYSIOLOGY

(Choice D) A mosaic pattern of lamellar bone with irregular sections of lamellar bone, linked by cement lines is seen in patients with Paget’s disease of the bone. The cement lines represent previous areas of bone resorption. (Choice E) Osteopetrosis, also known as “marble bone disease” is caused by decreased osteoclastic bone resorption. Histologically, osteopetrosis is characterized by the persistence of primary; un mineralized spongiosa in the medullary canals. In normal individuals bone marrow replaces the primary spongiosa. Educational Objective: Subperiosteal thinning is a characteristic feature of hyperparathyroidism. Radiologically, this thinning appears as subperiosteal erosions in the medial sides of the second and third phalanxes of the hand, and as a granular, “salt-and pepper” appearance of the calvarium.

256

USMLE WORLD STEP 1

PATHOPHYSIOLOGY

Q NO 57: An infant with ambiguous genitalia is born to a 26-year-old woman following an uncomplicated pregnancy. During the first week of life the infant develops hypotension and hyperkalemia. Cytogenetic studies reveal a46XX karyotype. The enzyme deficient in this patient is normally responsible for which of the following transformations? Cholesterol — Pregnenolone B. Progesterone — 17-hydroryprogesterone C. Testosterone — Dihydrotestosterone D. Progesterone — 11-Deoxycorhcosterone E. Androstenedione — Testosterone

A.

Explanation: Congenital adrenal hyperplasia (CAH) can result when there is a deficiency of an enzyme of cortisol and/or aldosterone synthesis. CAH is a heterogenous group of disorders because different enzymes may be involved. All types of CAH are inherited in an autosomal recessive fashion. In 90% of cases 21-hydroxIase is the deficient enzyme. 21-hydroxlase catalyzes both 11-deoxjcorticosterone and 11-deoxqcortisol synthesis. 11-deoxjcorticosterone is the precursor of corticosterone, which is the precursor of aldosterone, and 11-deoxytortisol is the precursor of cortisol. In 21hydroxjlase deficiency mineralocorticoid and glucocorticoid synthesis is decreased. Upstream precursors accumulate as a result, and channel into testosterone synthesis

257

USMLE WORLD STEP 1

PATHOPHYSIOLOGY

instead. Increased testosterone levels lead to ambiguous genitalia in girls (virilization). Male children have normal genitalia. Salt-wasting occurs due to the lack of mineralocorticoids. Affected infants often have recurrent vomiting, dehydration, hypotension, hyponatremia and hyperkalemia. Deficiencies of 11hydroxlase and 17-hydroxlase can also cause CAH.

(Choice A) Desmolase catalyzes the conversion of cholesterol into pregnenolone. Absence of this enzyme would lead to a complete absence of steroid hormones. An infant with a total lack of desmolase would not survive. (Choice B) Synthesis of 17-hydroxyprogesterone from progesterone is catalyzed by 17-hydroxilase. Deficiency of this enzyme leads to mineralocorticoid excess that manifests with salt retention and hypertension. Patients are phenotypically female. (Choice C) Dihydrotestosterone (DHT) is a biologically active metabolite of testosterone formed by 5cc-reductase in the prostate gland testes. Hair follicles and adrenal glands. Deficiency of (or insensitivity to) DH Tin utero is a cause of androgen insensitivity syndrome. (Choice E) Androstenedione is produced in the adrenals and the gonads as an intermediate in the synthesis of testosterone estrone and estradiol. Impaired androstenedione synthesis would lead to decreased levels of sex hormones. Educational Objective: 21-hydroxylase deficiency is the most common form of congenital adrenal hyperplasia. Affected female infants have ambiguous (virilized) genitalia. Saltwasting (vomiting hypotension and hyperkalemia) occurs due to mineralocorticoid deficiency.

258

USMLE WORLD STEP 1

PATHOPHYSIOLOGY

Q NO 58: A 32-year-old Asian female presents to your office complaining of easy fatigability and exertional dyspnea. Cardiac auscultation reveals an extra diastolic sound and a diastolic murmur at the apex. You suspect mitral stenosis. Cardiac catheterization findings are given on the slide below. Which of the following corresponds to the opening snap (OS) timing in this patient?

A. B. C. D. E.

A B C D E

Explanation: The opening snap (OS) in patents with mitral stenosis is an early diastolic sound due to tensing of the abnormal mitral valve (MV) leaflets after the valve cusps have completed their opening excursion. This occurs shortly after the mitral valve opens, when left ventricular pressure drops below left atrial pressure. From the graph above, it can be seen that the higher the early diastolic left atrial pressure the closer the opening snap will tend to be to point B, which corresponds to the A2 component of the heart sound. The A2-OS interval is inversely correlated with the severity of mitral stenosis. The more severe the stenosis, the higher the steady state left atrial pressure in early diastole and the shorter the A2-OS interval. (Choice A) This point corresponds to the opening of the aortic valve at the end of isovolumetric ventricular contraction in early systole, when the left ventricular pressure exceeds the aortic pressure. (Choice B) Point B corresponds to the closure of the aortic valve at the end of systole, producing the A2 component of the second heart sound. The aortic valve closes as soon as left ventricular pressure drops below the aortic pressure. (Choice D) This is a random point in diastole, when the mitral valve is already maximally open and ventricular filling is progressing. Note that there is a significant gradient of pressure between the left atrium and left ventricle during diastole, consistent with mitral stenosis. Under normal conditions, diastolic left atrial and left ventricular pressures are nearly equal. (Choice E) Point E roughly corresponds to the onset of atrial contraction during late ventricular diastole. The mitral valve is still maximally open. Note the increase in the gradient of pressure between the left atrium and left ventricle produced by atrial contraction. This can cause presystolic accentuation of the diastolic murmur of mitral stenosis. Educational Objective: Patients with mitral stenosis due to abnormal mitral valves may have an opening snap (OS) in early diastole, shortly alter the aortic component of the second heart sound (A2). The OS occurs shortly after the mitral valve opens.

259

USMLE WORLD STEP 1

PATHOPHYSIOLOGY

Q NO 59: A 12-year-old Caucasian male is found to have a wide, fixed splitting of the second heart sound (82) on routine physical examination. He denies any symptoms. If present, the congenital heart disease in this patient may require surgical repair to prevent irreversible changes in the: A. Right ventricle B. Right atrium C. Left ventricle D. Left atrium E. Pulmonary vessels F. Coronary vessels Explanation: Wide fixed splitting of S2 that does not vary with respiration is a characteristic auscultatory finding of an atrial septal defect (ASD). ASD creates a left-to-right shunt because of the high pressure in the left atrium. The result is increased blood flow through the pulmonary artery. The muscular pulmonary arteries may develop laminated medial hypertrophy that can become so severe overtime as to increase the pulmonary vascular resistance above the total systemic vascular resistance. At this point the original left-to-right intracardiac shunt reverses, and flow becomes right-to- left. This switch to right-to-left shunting manifests as late-onset cyanosis, with clubbing and polycythemia. Eisenmenger syndrome is the name for reversal of shunt flow through a congenital cardiac defect that occurs as a result of chronic pulmonary hypertension. Overtime the pulmonary vascular sclerosis becomes irreversible and closure of the cardiac septal defect can no longer be hemodynamically tolerated by the right ventricle. (Choices A and B) Pulmonary hypertension produced by an ASD could result in right ventricular hypertrophy and right atrial enlargement. However, these changes are not necessarily irreversible. If the pulmonary hypertension is corrected, the right heart can revert to a more normal morphology. (Choices C and D) Left ventricular failure is uncommon in patients with ASD. Left atrial enlargement can be present due to volume overload but the main changes are in the right side of the heart. Educational Objective: Wide fixed splitting of the second heart sound is a characteristic auscultatory finding in patients with ASD. A hemodynamically significant ASD can produce chronic pulmonary hypertension as a result of left-to-right intracardiac shunting. Eisenmenger syndrome is the late-onset reversal of a left-to-right shunt due to pulmonary vascular sclerosis resulting from chronic pulmonary hypertension. Closure of the ASD may be required to prevent irreversible pulmonary vascular sclerosis and a permanent Eisenmenger syndrome.

260

USMLE WORLD STEP 1

PATHOPHYSIOLOGY

Q NO 60: A 2-year-old female is undergoing evaluation for ambiguous genitalia. The patient has high blood pressure on three consecutive visits as well as marginally low serum potassium levels. Which of the following enzymes is most likely to be deficient in this patient? A. 17-hydroxqlase B. 21-hydraqlase C. 11-hydroxqlase D. Desmolase E. 5-alpha-reductase

Explanation: Congenital adrenal hyperplasia (CAH) is a group of disorders that result from defects in the enzyme pathway for cortisol biosynthesis in the adrenal gland. CAH can occur as a result of any of five enzymatic defects (see steroidogenic pathway): 1. 17-hydroxylase 2. 21-hydroxylase (most common) 3. 11-hydroxylase 4. 20, 22-desmolase

261

USMLE WORLD STEP 1

PATHOPHYSIOLOGY

5. 3-3-hydroxysteroid dehydrogenase 11-hydroxylase converts deoxycorticosterone to corticosterone and 11-deoxycortisol to cortisol. When there is a deficiency of this enzyme the adrenal gland cannot synthesize cortisol efficiently. Low cortisol levels stimulate pituitary production of ACTHI which increases the production of adrenal androgens because the high levels of cortisol precursors are diverted towards the adrenal androgen biosynthetic pathway (see diagram). Therefore, female patients with 11-hydroxylase deficiency develop ambiguous genitalia. Due to the block in aldosterone synthesis, deoxycorticosterone is increased. Deoxycortisone has mineralocorticoid activity, which leads to the development of low-renin hypertension. (Choice A) 17-hydroxylase deficiency accounts for less than 1% of patients with CAH. 17-hydroxylase converts pregnenolone to 17-hydrox’ypregnenolone and progesterone to 17-hydroxyprogesterone. This enzymatic pathway is active in both the adrenal glands as well as the gonads. Decreased cortisol production leads to increased levels of ACTH thus stimulating the formation of deoxycorticosterone and corticosterone by the adrenals and causing low-renin hypertension and hypokalemia. In females, the genitalia are normal at birth. However, these females have delayed puberty (no production of sex steroid) and are hypertensive (excessive production of deoxycorticosterone). Males are under virilized and hypertensive. (Choice B) 21-hydroxylase deficiency is the most common type of CAH accounting for 90% of patients. The enzyme 21-hydroxylase is responsible for converting 17hydroxyprogesterone to 11-deoxycortisol in the zona fasciculata and progesterone to deoxycorticosterone in the zona glomerulosa. Patients with 21-hydroxylase deficiency will have features of androgen excess cortisol deficiency and mineralocorticoid deficiency (hypotension). (Choice D) 2022-desmolase converts cholesterol to pregnenolone. Because this is the first enzyme in the steroidogenic pathway, the formation of all steroid hormones is affected. Cholesterol and cholesterol esters accumulate in the adrenal glands. (Choice E) 5-α-reductase deficiency causes ambiguous genitalia in males, as a result of the defective conversion of testosterone to dihydrotestosterone. This is not a form of CAH because cortisol production is not affected. Females with 5-a-reductase deficiency do not have ambiguous genitalia. Educational Objective: 11-hydroxylase deficiency typically results in excessive adrenal androgen and mineralocorticoid production.

262

USMLE WORLD STEP 1

PATHOPHYSIOLOGY

Q NO 61: Bone mass is an important pathogenetic determinant of fracture risk. On the graph below, bone mass versus lifetime curves are shown for two healthy, fracture-free women. Which of the following factors is most consistent with curve A?

A. Low body mass index (BMI) B. Smoking history C. Black race D. Glucocorticoid therapy E. Early menopause

Explanation: Statistically black females have higher bone density than Caucasian females. The reasons that bone mineral density is generally higher in people of African descent are not entirely understood. It is known that adipose tissue serves as a site of extraovarian estrogen production: it has been postulated that perhaps black females have more adipose tissue (fat) than white females. However adipose tissue cannot account entirely for the differences, as multivariate analyses have shown higher bone mass in black females even after adjustment for BMI. To confound matters even more, the risk of fracture in black females is significantly lower than white females at a given bone density. (Choice A) Bone density increases with increasing BMI. A bodyweight of less than 127 pounds is a risk factor for low bone density and fragility fractures. Decreased bone mineral density in low body weight females maybe related to lower estrogen levels, as there is less adipose tissue to produce estrogen. (Choice B) Current smoking is a major risk factor for osteoporosis and for osteoporotic fractures. Smokers have significantly lower bone mineral density compared to nonsmokers, a fact attributed to the antiestrogenic effect of smoking. This decrease in estrogen also allows smokers to have a lower incidence of fibrocystic breast disease and uterine cancer. This cancer reduction should not be an implied incentive to smoke. The overall health effects of smoking are resoundingly negative. (Choice D) Chronic glucocorticoid use is associated with lower bone mineral density for several reasons. Glucocorticoids decrease the gastrointestinal absorption of calcium, inhibit collagen synthesis by osteoblasts, decrease gonadotrophin releasing hormone (leading to hypogonadism), and increase urinary calcium loss. Despite these adverse effects, life would be simply miserable or nearly impossible for some patients without chronic steroid therapy. In these patients, glucocorticoid-induced bone loss is treated by optimization of calcium and vitamin D intake and by the use of bisphosphonates. (Choice E) Menopause causes accelerated bone loss due to a decrease in estrogen. Estrogens have an anabolic effect on bone by increasing osteoblastic activity and decreasing osteoclastic activity. Educational Objective: Remember the risk factors for osteoporosis: 1. Smoking 2. Menopause 3. Corticosteroid therapy 4. Physical inactivity 5. Caucasian race (immutable) 6. Low total body weight 7. Alcohol use

263

USMLE WORLD STEP 1

PATHOPHYSIOLOGY

Q NO 62: A mutation affecting the trypsinogen molecule has recently been described. The mutation affects the inactivating cleavage site of the enzyme. Patients having this abnormality would most likely suffer from: ulcer B. Liver cirrhosis C. Pancreatitis D. Megaloblastic anemia E. Gastric cancer

A. Peptic

Explanation: All pancreatic enzymes (except amylase and lipase) are synthesized and secreted in inactive form to protect the pancreas from autodigestion. These inactive proenzymes are then activated by trypsin in the duodenal lumen. Trypsin is produced from its own precursor, trypsinogen, by the action of duodenal enterokinase. Activated trypsin activates other trypsinogen molecules as well as the other proteolytic enzymes. The body has protective mechanisms to prevent trypsin from being prematurely activated. Pancreatic secretory trypsin inhibitor (PSTI) secreted by pancreatic acinar cells inhibits any trypsin abnormally activated within the pancreas. This protein prevents trypsin-mediated activation of other proteolytic enzymes and autodigestion of pancreatic tissue. When the inhibitory capacity of PSTI is exceeded by large amounts of active trypsin, trypsin itself can serve as its own inhibitor. Hereditary pancreatitis is a rare disorder that results from a mutation in the gene that encodes the trypsinogen molecule. This mutation leads to the synthesis of abnormal trypsinogen that is not susceptible to inhibition by either trypsin or PSTI. Patients experience recurrent attacks of acute pancreatitis. (Choices A and E) Peptic ulcer disease and gastric adenocarcinoma are strongly associated with H. pylon infection. (Choice B) Liver cirrhosis can be caused by any number of conditions, with alcoholism and chronic hepatitis B and C being the most common. There is no association with mutations of the trypsinogen gene. (Choice D) Megaloblastic anemia is caused by deficiencies of vitamin B 12 and/or folate. It is also associated with a number of drugs (such as methotrexate and phenytoin) and some inborn metabolic errors. Educational Objective: Trypsin activates all proteolytic pancreatic enzymes including it self. If activated prematurely (i.e. before reaching the duodenal lumen), it can cause autodigestion of the pancreatic tissue. A number of inhibitory mechanisms exist to prevent premature activation of trypsin. A gene mutation that renders trypsinogen insensitive to inhibition causes hereditary pancreatitis.

264

USMLE WORLD STEP 1

PATHOPHYSIOLOGY

Q NO 63: A 74-year-old female presented to the ER with the sudden onset of severe upper back pain. Physical examination shows point tenderness over the thoracic vertebra. Neurological examination is unremarkable. X-ray of the spine reveals a compression fracture of a thoracic vertebra. On the graph, area “C” shows the normal relationship between the serum concentrations of calcium and parathyroid hormone. Which of the following areas represent this patient’s metabolic state?

A. Area A B. Area B C. Area C D. Area D E. Area E

Explanation: Osteoporosis is characterized by decreased bone strength resulting from low bone mass and micro-architectural deterioration of bone tissue. The presence of a fragility fracture—a fracture due to minimal trauma such as after a fall from a standing height—is strongly suggestive of osteoporosis. The World Health Organization (WHO) has established a definition of osteoporosis based on bone measurement using dual x-ray absorptiometry. The WHO defines osteoporosis in postmenopausal females as a bone density that is 2.5 standard deviations below the mean bone mineral density for young adult women. There are two different types of bones in the body. The cortical or compact bone makes up the shafts of long bones and outer envelopes of all bones. Postmenopausal osteoporosis typically involves cancellous bone, which is predominantly present in the vertebral column, distal radius, hip, and neck of femur. In primary osteoporosis, serum calcium and PTH are typically in the normal range. (Choice A) Area A in the diagram reflects high serum PTH with low serum calcium. This is usually seen in patients with renal failure or with vitamin D deficiency. (Choices B and E) Area B reflects primary hyperparathyroidism. Area E shows high calcium levels in the presence ot low PTH levels, seen in patients with PTHindependent causes of hypercalcemia. The increase in serum calcium suppresses the secretion of PTH from normal parathyroid glands. Important causes of PTHindependent hypercalcemia include the humoral hypercalcemia of malignancy, vitamin D toxicity, excessive of ingestion of calcium, thyrotoxicosis, and immobilization. (Choice D) In patients with hypoparathyroidism, low PTH levels cause hypocalcemia. Educational Objective: Patients with osteoporosis have low bone mass, resulting in increased susceptibility for fragility fractures. In primary osteoporosis (osteoporosis not caused by a medical disorder), serum calcium, phosphorus, and PTH levels are typically normal.

265

USMLE WORLD STEP 1

PATHOPHYSIOLOGY

Q NO 64: A 58-year-old Caucasian male with severe exertional angina is found to have hypokinesia of the anterior wall of the left ventricle and a left ventricular ejection fraction of 35% on transthoracic echocardiography. His past medical history is significant for hypertension, diabetes and hypercholesterolemia. The patient undergoes coronary artery bypass grafting (CABG) and his condition improves significantly: hypokinesia is no longer evident and the ejection fraction increases to 50%. This phenomenon is referred to as: A. Ischemic preconditioning B. Myocardial stunning C. Hibernating myocardium D. Reperfusion injury E. Ventricular remodeling Explanation: Repetitive ischemia of cardiac myoc4es or persistent hypoperfusion of myocytes can result in a chronic but reversible loss of contractile function, referred to as “hibernation.” In hibernating myocardium, myocardial energy metabolism is reduced but there is sufficient ATP to prevent contracture. There maybe increased gene expression of TNF-α and NOS (inhibitors of contraction), complex defects in calcium cycling and excitation-contraction coupling, and/or decreased beta-adrenergic receptor density. The syndrome of cardiac muscle hibernation can be reversed by revascularization with CABG surgery or balloon angioplasty. The time-course of contractile function recovery is variable, occurring within hours to months. (Choice A) Ischemic preconditioning is the development of resistance to infarction by cardiac myocytes exposed to prior repetitive non-lethal ischemia. Sublethal ischemic insults can protect the affected myocardium against subsequent greater ischemic insults. (Choice B) Myocardial stunning is a less severe form of ischemia-induced reversible loss of contractile function than hibernation. Brief ischemic episodes (c 30 minutes) followed by reperfusion can produce myocardial stunning, and full recovery is usually over hours to days. In contrast, the patient in the vignette appears to have had a chronic depression of myocardial contractility. Repetitive stunning can result in hibernation. (Choice D) The patient in the vignette experienced a net improvement in myocardial contractile function with reperfusion; thus reperfusion injury was not the predominant process here. Reperfusion injury is further myocardial cell damage beyond that associated with the initial ischemic event. Proposed mechanisms of reperfusion injury include membrane damage from oxygen free radicals generated by infiltrating leukocytes during reperfusion, intracellular calcium overload, and apoptosis. (Choice E) Ventricular remodeling involves chronic changes in the mass, volume, shape, and myocyte composition of the heart, as a means of compensating for an increased hemodynamic load. Normal myocardium adjacent to ischemic, dysfunctional myocardium would be more likely to undergo remodeling than the ischemic myocardium itself. In any case, the rapidly of this patient’s improvement following revascularization indicates that the previously ischemic myocardium most likely resumed contraction: remodeling would take significantly longer to exert any significant effect. A persistent or repetitive low flow state causes myocardial hibernation that can be reversed by reperfusion. Myocardial stunning is a less severe form of ischemiainduced reversible loss of contractile function.

266

USMLE WORLD STEP 1

PATHOPHYSIOLOGY

Repetitive stunning can result in hibernation. Ischemic preconditioning is the development of resistance to infarction by cardiac myocytes previously exposed to repetitive non-lethal ischemia. Ventricular remodeling involves chronic changes in mass volume. Shape and myocyte composition of the heart, to compensate for an increased hemodynamic load.

267

USMLE WORLD STEP 1

PATHOPHYSIOLOGY

Q NO 65: A 72-year-old Caucasian male has a presystolic sound on cardiac auscultation that immediately precedes the first heart sound and is best heard when the patient turns to his left side and holds his breath. The patient’s blood pressure is 150/90 mmHg and his heart rate is 74 beats per minute and regular. He has a long history of hypertension and evidence of extensive calcinosis around the mitral and aortic valves on chest x-ray. The extra sound is most likely due Increased flow velocity through the aortic valve B. Restricted motion of the aortic valve cusps C. Restricted motion of the mitral valve cusps D. Papillary muscle tension after the rapid filling of the ventricles E. Increased stiffness of the left ventricular wall

A.

Explanation: An S4 gallop (also known as an atrial sound or atrial gallop) is a presystolic sound on cardiac auscultation that immediately precedes S1. A left-sided S4 is heard best at the cardiac apex with the patient in the left lateral decubitus position, and a rightsided S4 is heard best along the lower left sternal border (the tricuspid area) with the patient in the supine position. An S4 is heard when there is a sudden rise in end diastolic ventricular pressure caused by atrial contraction against a ventricle that has reached the limit of its compliance. Thus, an S4 may be present in any condition that causes a stiff ventricle. Degenerative mitral annular calcification or aortic valve calcification can be associated with chronically elevated LV pressures and systemic hypertension. This patient probably has left ventricular hypertrophy (LVH) associated with hypertensive heart disease. LVH reduces ventricular compliance and can cause diastolic dysfunction. A normal atrial contraction is required to generate an S4. (Choice A) This would tend to produce the murmur of aortic stenosis, an ejectiontype murmur that occurs during systole (after S1 and before A2). (Choice B) Restricted motion of the aortic valve cusps might be associated with an aortic ejection click, aortic stenosis, and/or aortic regurgitation. In any event, the extra sound(s) produced would occur after S1, during systole. (Choice C) Restricted motion of the mitral valve cusps could result in abnormal diastolic sounds, like an opening snap and/or a murmur of mitral stenosis (MS). The opening snap would occur early in diastole. Pre-systolic accentuation (due to atrial contraction) of an otherwise inaudible murmur of MS could explain the extra sound heard in this patient. However, isolated MS is generally associated with normal or reduced left ventricular pressures making degenerative mitral annular calcification as seen in this patient less likely. (Choice D)The papillary muscles are not placed under increased tension during diastolic ventricular filling. They are tensed during ventricular systole, after S1. Educational Objective: A presystolic sound on cardiac auscultation that immediately precedes S1 is most often an S4 gallop. An S4 requires normal atrial contraction and results from rapid emptying of atrial blood into a ventricle with reduced compliance (stiff ventricle).

268

USMLE WORLD STEP 1

PATHOPHYSIOLOGY

Q NO 66: A 30-year-old female with type 1 diabetes is being evaluated for anorexia, weight loss and decreased insulin requirement. Physical examination shows generalized hyperpigmentation. An ACTH stimulation test fails to elicit an increase in her serum cortisol levels. Which of the following set of laboratory values is most likely present in this woman?

Explanation: The patient described in the vignette has features of primary adrenal insufficiency. As a patient with type 1 diabetes mellitus (a disorder that results from the autoimmune destruction of pancreatic β-cells), she is prone to develop other autoimmune disorders, including autoimmune adrenalitis. Autoimmune adrenalitis is a condition where all three zones of the adrenal cortex are attacked by the body’s immune system, resulting in primary adrenal insufficiency (“primary” because the defect is inherent to the adrenal itself). In primary adrenal insufficiency, administration of exogenous ACTH does not increase serum cortisol levels. Autoimmune adrenalitis typically spares the adrenal medulla. Typical biochemical abnormalities in patients with primary adrenal insufficiency include hyponatremia, hyperkalemia, hypochloremia, and metabolic acidosis. In patients with secondary and tertiary adrenal insufficiency (due to defects in the pituitary and hypothalamus, respectively), a deficiency of mineralocorticoid does not usually occur. Thus, these patients do not develop hyperkalemia or metabolic acidosis. Patients with secondary and tertiary adrenal insufficiency will typically respond with a serum cortisol increase when exogenous ACTH is administered. (Choices A and D)These choices show biochemical abnormalities characteristic of mineralocorticoid (i.e. aldosterone) excess. Elevated aldosterone levels lead to sodium retention, potassium loss, and bicarbonate retention. Thus, hypokalemia and metabolic alkalosis are common in this condition. Hypernatremia, however, does not usually occur, due to the phenomenon of “aldosterone escape,” whereby initial increases in sodium and chloride resorption lead to intravascular hypervolemia, which in turn causes release of atrial natriuretic peptide, leading to diuresis and compensatory sodium loss. (Choice C) The finding of hyponatremia, hypokalemia, and hypochloremia with metabolic alkalosis suggests diuretic use. Diuretics lead to urinary loss of sodium, potassium and chloride, while promoting increased bicarbonate absorption secondary to volume contraction (“contraction alkalosis”). (Choice E) Normal serum sodium in the setting of hypokalemia and metabolic acidosis is suggestive of renal tubular acidosis (RTA). Educational Objective: Patients with type 1 diabetes mellitus are prone to develop other autoimmune endocrinopathies, including Hashimoto thyroiditis, Graves disease, and Addison’s disease (hypoadrenocorticism). These patients are at increased risk for other non-endocrine autoimmune disorders as well, including vitiligo and pernicious anemia.

269

USMLE WORLD STEP 1

PATHOPHYSIOLOGY

Q NO 67: A 33-year-old Hispanic female who recently emigrated from Mexico is brought to the ER with severe shortness of breath and hemoptysis. She is A. treated with diuretics, and begins to feel better. However, she develops rightsided hemiparesis soon after. Based on the history and initial physical examination, you suspect mitral stenosis is responsible. Which of the following findings suggests an associated lesion or another diagnosis in this patient? Right ventricular dilation B. Increased systolic pulmonary artery pressure C. Increased pulmonary capillary wedge pressure D. Tricuspid regurgitation E. Increased diastolic left ventricular pressure F. Reduced pulmonary compliance Explanation: In isolated mitral stenosis (MS) diastolic pressures in the left ventricle (LV) are usually near normal. Only pressures proximal to the stenotic mitral valve would be markedly elevated. When a patient with suspected MS is also found to have an increased LV end diastolic pressure, the presence of an additional lesion is likely. Possibilities include rheumatic involvement of the aortic valve (which typically causes combined aortic stenosis and regurgitation), or infective endocarditis superimposed on an aortic valve deformed by chronic rheumatic heart disease (RHD). In patients with RHDI the mitral valve alone is the sole site of involvement in 65% to 70% of cases. Both the mitral and aortic valves are affected in about 25% of cases. This patient’s right-sided hemiparesis may have arisen from an embolic stroke complicating isolated MS (via atrial dilatation, atrial mural thrombosis and thromboembolism) or from an aortic valve vegetation generated by endocarditis. (Choice A) Right ventricular dilatation is found in mitral stenosis that is severe enough to cause significant pulmonary hypertension. (Choice B) This finding would be expected in isolated mitral stenosis severe enough to cause significant pulmonary hypertension via backward transmission of elevated left atrial pressure. (Choice C) This finding would be expected in isolated mitral stenosis (MS) severe enough to cause a significant elevation of left atrial pressure. A properly measured pulmonary capillary wedge pressure reflects the left atrial transmural pressure at end diastole, which is elevated in MS, even though left ventricular end diastolic transmural pressure (LVEDP) may be normal. Note that the wedge pressure does not reflect LVEDP in patients with MS. (Choice D) Tricuspid regurgitation may occur as a complication of severe isolated mitral stenosis, as a consequence of right ventricular dilatation due to pulmonary hypertension. (Choice F) Reduced pulmonary vascular compliance may complicate long-standing pulmonary hypertension induced by isolated mitral stenosis. Pulmonary vascular endothelial dysfunction and reactive vasoconstriction can result from pulmonary hypertension. Chronic pulmonary hypertension can also cause reactive hypertrophy in the walls of pulmonary vessels (pulmonary vascular sclerosis). Both of these processes reduce pulmonary vascular compliance. Educational Objective: Isolated mitral stenosis (MS) elevates the left atrial end diastolic pressure and can therefore cause an elevated pulmonary capillary wedge pressure (PCWP) pulmonary hypertension, pulmonary vascular sclerosis and decreased compliance, right ventricular dilatation, and functional tricuspid regurgitation. Note that in patients with isolated MS, PCWP does not reflect the left ventricular end diastolic pressure which maybe normal.

270

USMLE WORLD STEP 1

PATHOPHYSIOLOGY

Q NO 68: A 46-year-old Caucasian female presents to your office because of easy fatigability and exertional dyspnea. Auscultation of the heart reveals a diminished first heart sound and an apical holosystolic murmur radiating to the axilla. Lungs have bibasilar crackles. There is no elevation of jugular venous pressure or peripheral edema. Which of the following would most likely increase forward-toregurgitant volume ratio in this patient? A. Decreasing left ventricular preload B. Increasing left ventricular contractility C. Decreasing left ventricular afterload D. Decreasing heart rate E. Increasing left ventricular volume Explanation: This patient has clinical features suggestive of mitral regurgitation with left sided heart failure. In a patient with mitral regurgitation, some of the blood in the left ventricle is pumped forward through the aortic valve and is considered the forward stroke volume (FSV) while some is forced backwards through the incompetent mitral valve into the left atrium and is considered the regurgitant stroke volume (RSV). If the systolic retrograde flow resistance between the LV and LA, the preload (LV end diastolic volume) and the contractility remain the same, the amount of blood that flows forward and backward is determined by the left ventricular afterload (or systolic intraventricular pressure). The lower the average LV afterload, the lower will be the average systolic pressure gradient driving regurgitant flow into the LA and the lower will be the RSV. Moreover, FSV will be increased. This will increase the forward-to-regurgitant volume ratio. Thus arterial vasodilator therapy, which acts to decrease LV afterload, tends to decrease RSV and increase FSV in patients with MR. (Choice A) LV preload reduction might decrease the regurgitant flow fraction if the degree of mitral valve incompetence was LV end diastolic volume dependent (eg. in dilated cardiomyopathy) or if the reduction in LV preload was also accompanied by a slight decrease in LV afterload. However these effects on the regurgitant flow fraction would generally be quantitatively less than that of a significant reduction of afterload. (Choice B) For a fixed anatomical degree of mitral valve incompetence producing mitral regurgitation (MR) and at a given left ventricular preload, an increase in LV contractility would tend to increase LV afterload. The latter would tend to increase the fraction of each LV stroke volume going into regurgitant flow. Thus an isolated increase in LV contractility would tend to increase MR. Partly for this reason some experts recommend chronic beta blocker therapy for patients with significant MR. (Choice D) A primary decrease in heart rate (HR) would tend to increase LV end diastolic volume (preload). In the new steady state after recruitment of cardiovascular reflexes to maintain forward LV output and mean systemic arterial pressure LV afterload might not be significantly changed. Moreover in some cases of MR where LV dilatation alone maybe responsible the fraction of regurgitant flow can increase as the LV dilates further (as preload is increased). Under these conditions a primary decrease in heart rate could decrease the forward to regurgitant LV output ratio. (Choice E) Presumably this choice means increasing LV end diastolic volume or preload. As explained above increasing LV preload does not significantly alter steady state LV afterload, the major determinant of the degree of mitral regurgitation. Moreover LV dilatation alone can contribute to or worsen MR.

271

USMLE WORLD STEP 1

PATHOPHYSIOLOGY

Educational Objective: The major determinant of the ratio of forward-to-regurgitant left ventricular (LV) output in patients with mitral regurgitation (MR) is LV afterload. An increase in steady state LV afterload tends to decrease this ratio. An increase in steady state LV end diastolic volume may contribute to or worsen MR.

272

USMLE WORLD STEP 1

PATHOPHYSIOLOGY

Q NO 69: A 60-year-old Caucasian male presents to your office for a routine checkup. He has no complaints. His blood pressure is 165/110 mmHg, and heart rate is 75/mm. Physical examination reveals a moderately overweight man (BMI = 28.3 kg/m2) with predominantly central fat distribution. Lab work demonstrates a blood glucose level of 150 mg/dL. If present, which of the following is most likely responsible for the resistance of peripheral tissues to the action of insulin in this patient? A. High low density lipoprotein (LDL) B. Low high density lipoprotein (HDL) C. High free fairy acids (FFA) D. High serum C-peptide level E. High serum beta-hydroxybutyrate F. High homocysteine Explanation: One of the important actions of insulin is the facilitation of glucose uptake by adipocytes and muscle cells. Several genetic and environmental factors can cause insulin resistance. High free fa1ti acid levels are one environmental factor that results in insulin resistance. The mechanism by which free fatty acid induces insulin resistance is unclear. Serine phosphorylation of the insulin receptor’s beta subunit could be involved. This phosphorylation of serine interferes with down-stream signaling because serine kinase, instead of tyrosine kinase, becomes activated. Serine phosphorylation is a known mechanism of insulin resistance induced by TNEalpha, glucagon, and glucocorticoids. Free fatty acids also act to decrease insulin secretion, which prevents the compensatory rise of insulin that is required to overcome insulin resistance. The induction of insulin resistance and beta cell dysfunction along with high free fatty acids is termed “lipo toxicity.” Lowering free fatty acids improves beta cell function and insulin resistance. The other choices listed do not interfere with insulin secretion or insulin action. (Choices A and B) LDL is metabolized by receptor-mediated endocytosis, mainly in the liver. LDL levels are governed by diet, liver production, and receptor-mediated LDL uptake. Insulin is not directly involved in LDL metabolism. High serum triglycerides and free fatty acids are commonly seen in insulin resistance and uncontrolled diabetes mellitus. The surface phospholipids in LDL particles are replaced by triglycerides in diabetics. The subsequent removal of surface triglyceride in the liver by hepatic lipase results in highly atherogenic small dense LDL particles. Serum HDL is generally low in diabetics because the altered cell surface composition accelerates the clearance of HDL. In contrast to free fatty acids, LDL and HDL do not alter insulin signaling. (Choice D) C-peptide is secreted in equimolar amount with insulin from pancreatic beta cells. It is true that increased C-peptide levels are present in insulin resistance, but C-peptide does not have any biological effect. (Choice E) Beta hydroxybutyrate is a marker of insulin deficiency and would be present only in type 1 diabetes mellitus. Patients with type 2 diabetes mellitus do not have high beta hydroxybutyrate because they do not have absolute deficiency of circulating insulin. Beta hydroxybutyrates do not interfere with the actions of insulin. (Choice F) Homocystinemia has been linked to atherosclerosis. Lowering of homocysteine levels by folic acid treatment has been shown to decrease the progression of atherosclerosis in some studies. However, high homocysteine levels do not interfere with insulin action. Educational Objective: EEA and serum triglycerides are believed to increase insulin resistance in overweight individuals.

273

USMLE WORLD STEP 1

PATHOPHYSIOLOGY

Q NO 70: A 40-year-old Caucasian female is hospitalized by a femoral neck fracture after falling from a standing height—more specifically, she slipped in the tub. Three years ago, she was diagnosed with a prolactinoma, but she has refused treatment. The tumor has not significantly increased in size over the last two years nor has it caused any visual deficits. The pathophysiology underling this patient’s susceptibility to bone fracture is additionally responsible for which of the following? A. Stomach ulcers B. Vaginal dryness C. Kidney stones D. Hand tremor E. Swallowing difficulty Explanation: Prolactinomas are the most common pituitary tumors. In females prolactinomas typically cause amenorrhea and galactorrhea. Reproductive dysfunction occurs because high levels of prolactin suppress gonadotrophin-releasing hormone (GnRH) from the hypothalamus, leading to hypogonadism, anovulation, and amenorrhea. The direct effect of prolactin on the breasts causes milk discharge (galactorrhea). If hyperprolactinemia is not treated, prolonged hypogonadism (low estrogen) causes accelerated bone loss, exactly as occurs with postmenopausal osteopenia/osteoporosis. Low bone density puts patients at a high risk for fragility fractures (fractures sustained with minimal trauma such as fall from standing height, also called “pathological fractures”). Low bone density and vaginal dryness are very frequent sequelae of decreased estrogen. Other causes of accelerated bone loss include hyperparathyroidism, hyperthyroidism, chronic glucocorticoid excess, and vitamin D deficiency. All of these causes of low bone density should be ruled out in patients with significantly low bone density. (Choices A and C) Hyperparathyroidism can cause kidney stones, probably as a result of high serum calcium. Stomach ulcers can occur with isolated hyperparathyroidism or with the MEN-associated Zollinger-Ellison syndrome of excess gastrin. (Choices D and E) Hyperthyroidism causes hand tremors and an enlarged thyroid that can make it difficulty to swallow. Educational Objective: Low bone density is a well known complication of hyperprolactinemia that is associated with prolactin-induced hypogonadism (ie, low estrogen). Vaginal dryness is another very common manifestation of estrogen deficiency.

274

USMLE WORLD STEP 1

PATHOPHYSIOLOGY

Q NO 71: A male patient is being evaluated for progressive shortness of breath. His flow-volume curve is pictured below. Which of the following pathologic findings is most likely in this patient?

A. Pulmonary fibrosis B. Alveolar hyaline membranes C. Interalveolar wall destruction D. Intraalveolar hemorrhage E. Thromboembolic disease F. Compression atelectasis

Explanation: This patient’s flow-volume loop exhibits a marked reduction in expiratory flow rate compared to the normal loop. Moreover, the patient’s residual lung volume (RVI at end expiration) is increased and tidal volume is decreased compared to normal. These abnormalities are characteristic of chronic obstructive pulmonary disease (COPD). An obstructive flow-volume loop profile may result from increased bronchial resistance (e.g. due to anatomic narrowing as in chronic bronchitis) and/or from decreased lung elasticity (e.g. due to destruction of interalveolar walls as in emphysema). Decreased lung elasticity also promotes dynamic compression of otherwise normal airways during expiration, contributing to increased expiratory airflow resistance in bronchi. The expiratory airflow obstruction in chronic bronchitis and/or emphysema causes hyperinflation (increased RV) and decreased tidal volumes. (Choice A) Pulmonary fibrosis tends to decrease lung volume and compliance, producing a restrictive pattern in spirometric flow volume loops, as shown by the red curve below.

Note that in restrictive diseases, tidal volume and residual volume are reduced but the expiratory flow rate may be normal or only modestly reduced.

275

USMLE WORLD STEP 1

PATHOPHYSIOLOGY

(Choice B) Alveolar hyaline membranes tend to reduce lung compliance, causing a restrictive flow-volume pattern. (Choice D) Diffuse intraalveolar hemorrhage would tend to dilute alveolar surfactant, thus increasing alveolar surface tension and promoting alveolar atelectasis. These changes would reduce lung volumes and lung compliance. A restrictive flow-volume pattern would likely result. (Choice E) In thromboembolism, only perfusion is limited; there is no effect on ventilation. Therefore, pulmonary function tests would not be affected. (Choice F) Atelectasis due to extrinsic pulmonary compression may occur when part or all of the pleural cavity is filled by air (as in pneumothorax), fluid, exudate, and/or blood. Under such conditions, lung volumes and total lung compliance would likely be reduced, producing a restrictive flow-volume pattern on spirometry. Educational Objective: Reduced expiratory air flow velocity, decreased tidal volume, and increased residual volume together characterize chronic obstructive pulmonary disease (COPD). Chronic bronchitis and/or emphysematous destruction of interalveolar walls maybe responsible. In comparison, restrictive lung disorders tend to decrease all lung volumes without significantly affecting expiratory flow rates.

276

USMLE WORLD STEP 1

PATHOPHYSIOLOGY

Q NO 72: A 24-year-old male presents to your office complaining of paroxysmal episodes of breathlessness and wheezing. Sputum microscopy shows many granule-containing cells and crystalloid masses. The sputum findings suggest the role of which of the following substances in this patient’s condition? B. lL-3 C. lL4 D. lL5 E. y-Interferon F. TGF-3

A. lL1

Explanation: Paroxysmal breathlessness and wheezing in a young patient unrelated to ingestion of aspirin pulmonary infection, inhaled irritants, stress, and/or and exercise should raise a strong suspicion for extrinsic allergic asthma. The granule- containing cells in the sputum are most likely eosinophils and the crystalloid bodies are most likely Charcot-Leyden crystals (contain eosinophil membrane protein). Chronic eosinophilic bronchitis in asthmatics involves bronchial wall infiltration by numerous activated eosinophils, largely in response to IL-5 released by allergen-activated TH2 cells. (Choice A) Although macrophage IL-i release is involved in asthma pathogenesis, itis a not specific for this process. IL-i release is a component of almost all inflammatory processes, and does not necessarily cause eosinophil infiltration. (Choice B) IL-β functions mainly to promote the growth and differentiation of bone marrow stem cells. (Choice C) Release of IL-4 by TH2 type helper T-cells is a non-specific pathway involved in the stimulation of B-cell antibody production. IL- 4 is not chemotactic for eosinophils, however. (Choice E) y-Interferon is secreted by helper l-cells and functions to activate macrophages, thereby promoting adaptive immunity against intracellular pathogens. It is not directly chemotactic for eosinophils. (Choice F) Transforming growth factor beta (TGE-3) is growth factor involved in tissue regeneration and repair. Educational Objective: Paroxysmal breathlessness and wheezing in a young patient that is unrelated to ingestion of aspirin pulmonary infection, inhalation of irritants, stress, and/or and exercise should raise a strong suspicion for extrinsic allergic asthma. Classic sputum findings include eosinophils and Charcot-Leyden crystals. Eosinophils are recruited and activated bylL-5 secreted bVTH2 type helper l-cells.

277

USMLE WORLD STEP 1

PATHOPHYSIOLOGY

Q NO 73: A 56-year-old male presents to your office with progressive vision impairment. His past medical history includes diabetes mellitus, hypertension and gout. Physical examination suggests bilateral clouding of the lens in this patient. Which of the following metabolic conversions in the peripheral tissues most likely contributed to this patient’s current condition? Galactitol to sorbitol B. Glucose to sorbitol C. Fructose to galactitol D. Glucose to glyceraldehyde F. Adenine to uric acid

A.

Explanation: Complications of long-standing diabetes mellitus such as microangiopathy, retinopathy nephropathy and peripheral neuropathy occur at least in part due to chronic hyperglycemia that induces a number of metabolic changes. The most important mechanisms involved in the development of complications are: 1. Advanced glycosylation end products: Glycosylation refers to the attachment of glucose to amino acid residues in various proteins forming reversible glycosylation products that slowly stabilize to irreversible products. Glycosylation products accumulate and cross-links with collagen in blood vessel walls and interstitial tissues contributing to microangiopathy and nephropathy. Cross-linking of proteins by glycosylation products also facilitates inflammatory cell invasion and deposition of LDL in the vascular walls leading to atherosclerosis. 2. Polyol pathway impairment occurs in tissues that do not depend on insulin for glucose transport (lens, peripheral nerves, blood vessels and kidneys). Hyperglycemia results in increased intracellular glucose concentrations in these tissues. Glucose undergoes conversion into sorbitol (Choice B) by aldose reductase, and sorbitol, in turn, is converted into fructose. Sorbitol and fructose increase the osmotic pressure in tissues and stimulate the influx of water leading to osmotic cellular injury. Increased water in lens fiber cells leads to rupture of these cells with resultant opacification of the lens and cataract formation. Osmotic injury of Schwann cells contributes to peripheral neuropathy in diabetes. (Choice A) Aldose reductase coverts galactose into galactitol. If the level of galactose is increased, such as in galactosemia, more galactitol is produced. Elevated galactitol levels cause cataracts in patients with galactosemia. Synthesis of galactitol is not increased in DM. (Choice C) Galactitol is produced from galactose, not from fructose. (Choice D) In the hexose monophosphate pathway, glucose is converted to glyceraldehyde-3-phosphate that can enter glycolysis. This metabolic pathway does not play any role in development of diabetic complications. (Choice E) Deficiency of enzyme adenosine deaminase impairs the conversion of adenosine to uric acid. This enzyme is absent or defective in severe combined immunodeficiency (SCID). Educational Objective: During hyperglycemia, excess plasma glucose is converted to sorbitol by aldose reductase. Sorbitol accumulates within some cels and attracts water into these tissues leading to osmotic cellular injury. This mechanism is implicated in the pathophysiology of cataracts and peripheral neuropathy in diabetes.

278

USMLE WORLD STEP 1

PATHOPHYSIOLOGY

Q NO 74: A genetically female infant born to a 23-year-old female demonstrates ambiguous genitalia and a normal-sized uterus. Blood pressure is normal, but serum testosterone and androstenedione levels are increased. The mother experienced increased facial hair growth and some voice deepening during pregnancy. Which of the following is most likely deficient in the newborn? A. 21-hydroxylase B. Aromatase C. 17-hydroxylase D. 5α-reductase F. HMG CoA reductase Explanation: Aromatase is an enzyme that converts androstenedione to estrone and testosterone to estradiol. These metabolic steps are essential in estrogen synthesis and occur in numerous tissues including the ovari, testis placenta, brain, liver, muscle and adipose tissue. Aromatase deficiency manifests early in embryonal life with high androgen levels and low estrogen levels in the female fetus. This hormonal imbalance can affect the mother causing virilization during pregnancy due to the transfer of excess androgens into the maternal circulation via the placenta. At birth, affected female infants have ambiguous or male-type external genitalia (female pseudohermaphrodism). Females with aromatase deficiency have primary amenorrhea and tall stature because estrogen is necessary for fusion of the epi physes. Males with this defect exhibit tall stature and osteoporosis with no genital abnormalities. Increased aromatase levels, on the other hand, lead to estrogen excess causing gynecomastia in boys and precocious puberty in girls. Patients of both sexes have short stature due to early closure of the epiphyses. (Choice A) 21-hydroxqlase deficiency is the most common tqpe of congenital adrenal hyperplasia (CAH). It presents with ambiguous genitalia in the female fetus and salt wasting resulting in hypotension, hyponatremia and hyperkalemia. (Choice C) 1 7-hydroxylase deficiency is a rare form of congenital adrenal hyperplasia (CAH). Its symptoms include sodium retention hypertension, and ambiguous genitalia in the male fetus. (Choice D) Deficiency of 5a-reductase leads to decreased synthesis of dihydrotestosterone (DHT) in the male fetus. Affected infants have feminized external genitalia at birth, and experience virilization at puberty. (Choice E) HMG CoA reductase catalyzes the first step in the cholesterol synthesis pathway. It is the target of the statin class of lipid-lowering drugs. This enzyme is not involved in sexual differentiation. Educational Objective: Aromatase converts androgens into estrogens in the ovaries, testes, placenta and other peripheral tissues. Genetic deficiency of this enzyme leads to an inability to synthesize estrogens. It presents with maternal virilization during pregnancy and masculinization of the female fetus.

279

USMLE WORLD STEP 1

PATHOPHYSIOLOGY

Q NO 75: A 50-year-old Caucasian female presents to your office with periodic flatulence, abdominal pain and explosive diarrhea. After careful history taking, you suspect lactase deficiency as the cause of this patient’s symptoms. If your diagnosis is correct, which of the following is most likely to decrease on lactose challenge in this patient? Stool osmotic gap B. Stool pH C. Breath hydrogen content D. D-xylose absorption E. Cobalamin absorption

A.

Explanation: Congenital lactase deficiency is a rare autosomal recessive condition caused by a gene near (but separate from) the lactase gene, with affected newborns exposed to lactose experiencing explosive watery diarrhea. The more common form of lactose intolerance is acquired lactase deficiency, which occurs in 80-95% of Native Americans, 90% of some Asians, 65-75% of Africans and African Americans, and 50% of Hispanics: in these groups, symptoms appear by mid- childhood once intestinal lactase levels have plummeted. Lastly, some individuals appear to experience lactose malabsorption secondary to bacterial overgrowth or mucosal injury of the gastrointestinal tract. Lactase deficiency by any means causes incomplete hydrolysis of the disaccharide lactose into the monosaccharides glucose and galactose. The undigested lactose then accumulates within the gastrointestinal lumen, leading to osmotic diarrhea. Bacterial fermentation of lactose produces short-chain fatty acids and excess amounts of hydrogen. The presence of this hydrogen acidifies the stool, lowering its pH. The lactose tolerance test entails the oral administration of 50 g of lactose, with blood levels measured at 0, 60, and 120 minutes. If the blood glucose increases c 20 mg/dL and the individual experiences symptoms (eg, abdominal pain, bloating, flatulence, diarrhea, or vomiting), the diagnosis of lactose intolerance is confirmed. (Choice A) At its maximum, stool osmolality approaches serum osmolality (approximately 290 mOsm/kg). Most stool osmolality is accounted for by sodium and potassium, with colonic fermentation products composing the remainder (ie, the stool osmotic gap). When electrolytes comprise most of the luminal osmolality, the stool osmotic gap is low (c 50 mOsm/kg). When poorly absorbable substances are present, the stool osmotic gap is large (> 100 mOsm/kg). Therefore, the stool osmotic gap will be increased in lactase-deficient patients. (Choice C) Breath hydrogen content increases by more than 20 ppm in lactasedeficient patients and can be measured in exhaled air by gas chromatography. (Choice D) D-xylose absorption can be diminished in numerous malabsorptive conditions, including celiac sprue. It is not classically associated with lactose intolerance. (Choice E) Cobalamin (vitamin B 12) absorption is hindered in patients with pernicious anemia. It is not classically associated with lactose intolerance. Educational Objective: Lactase-deficient individuals have increased stool osmotic gap, increased breath hydrogen content, and decreased stool pH upon lactose challenge.

280

USMLE WORLD STEP 1

PATHOPHYSIOLOGY

Q NO 76: A 64-year-old male with uncontrolled diabetes and hypertension is evaluated for fatigue. His laboratory studies show a BUN of 75 mg/dL and creatinine of 5.8 mg/dL. On the graph, area “C” shows the normal relationship between serum concentrations of calcium and parathyroid hormone. Which of the following areas represent this patient’s metabolic state?

A. Area A B. Area B C. Area C D. Area D E. Area E

Explanation: Parathyroid hormone (PTH) is one of the principal regulators in keeping serum calcium within a tight normal range. PTH increases serum calcium by increasing osteoclastic bone resorption, increasing the distal tubular absorption of calcium. and increasing the formation of 1, 25-dihydroxvitamin D through up-regulation of the renal enzyme 1-alpha hydroxylase. 1, 25-dihydox vitamin D increases the gastrointestinal absorption of calcium. When PTH has adequately increased serum calcium, calcium-sensing receptors present on the parathyroid discourage the further secretion of PTH in a classic feedback loop. This patient is in renal failure, as demonstrated by his BUN and creatinine levels. The activity of renal 1-alpha hydroxylase is decreased in chronic kidney disease which causes the decreased formation of 125-dihyroxyvitamin D. The gastrointestinal absorption of calcium is consequently decreased; the ultimate result is a decrease in serum calcium. Another electrol4e change is the elevation of phosphorous because chronic renal disease impairs its excretion. The result of the decrease in 125-dihydroxjvitamin D the decrease in serum calcium, and the increase in serum phosphorus is an increase in the secretion of parathyroid hormone, a state termed “secondary hyperparathyroidism.” Despite excess PTH, calcium levels remain in the normal to only slightly low range; PTH is unable to increase the serum calcium because of the deficiency in 1-alpha hydroxylase. Area A on the graph demonstrates high PTH despite low calcium levels. (Choice B) Area B in this diagram reflects the effects primary hyperparathyroidism. Because the production of PTH is not subject to negative feedback in these patients, high calcium levels are unable to suppress the secretion of PTH. (Choice D) In patients with hypoparathyroidism, low PTH levels are accompanied by hypocalcemia and hyperphosphatemia. (Choice E) Area E shows high calcium levels in the presence of low PTH levels. This is seen in patients with PTH independent causes of hypercalcemia. Important causes of PTH-independent causes of hypercalcemia include the humoral hypercalcemia of malignancy, vitamin D toxicity, excessive ingestion of calcium, thyrotoxicosis, and immobilization (calcium is resorbed from inactive bones).

281

USMLE WORLD STEP 1

PATHOPHYSIOLOGY

Educational Objective: Secondary hyperparathyroidism is seen in patients with chronic renal failure. These patients have an elevated serum PTH accompanied by normal to low serum calcium levels, and high serum phosphorus levels. Circulating 1, 25 dihydroxy vitamin D levels are low due to the deficiency of 1-alpha hydroxylase, an enzyme that resides in the kidneys.

282

USMLE WORLD STEP 1

PATHOPHYSIOLOGY

Q NO 77: A 40-year-old female presented to the physician for evaluation of worsening fatigue. She has no other medical problems. She does not use tobacco, alcohol or drugs. Vital signs are within normal limits. Physical examination is within normal limits. Laboratory evaluation shows an alkaline phosphatase level of 180 U/L. Which of the following should be checked next? Unconjugated bilirubin level B. Lactate dehydrogenase level C. Serum ammonia levels D. Prothrombin time E. y-glutamyl transpeptidase F. Aspartate aminotransferase

A.

Explanation: Broadly speaking laboratory tests used in the evaluation of hepatobiliary disease either assess liver functionality (eg, prothrombin time bilirubin, albumin, cholesterol) or the structural integrity and cellular intactness of the liver (eg, transaminases) or the biliary tract (eg, alkaline phosphatase, y-glutaryl transferase). Alkaline phosphatase represents a group of enzymes associated with metabolic activity in a number of tissues including liver, bone, intestine, kidney, placenta leukocytes, and some neoplasms. Bone and liver are the primary sources of alkaline phosphatase with threefold elevation considered a relatively nonspecific finding for many liver diseases. To clarify the import of a moderately elevated alkaline phosphatase the hepatic y-glutamyl transpeptidase (GGTP) should be evaluated. Y glutamyl transpeptidase is an enzyme predominantly present in hepatocytes and biliary epithelia. While it too can be found in various extrahepatic tissues (organs such as kidney spleen pancreas heart, lung and brain) GGTP is not present to a significant extent in bone. It is therefore particularly useful in determining whether an elevated alkaline phosphatase is of hepatic or bony origin. (Choice A) Measurement of unconjugated bilirubin (especially in contrast to conjugated bilirubin) can be of assistance in distinguishing hemolytic conditions from hepatic dysfunction or bile duct obstruction. This test is not particularly helpful in following up an elevated alkaline phosphatase. (Choice B) Lactate dehydrogenase is a relatively nonspecific test that can be of some assistance in the evaluation of tissue injury or death. It is not the most helpful test in following up an elevated alkaline phosphatase. (Choice C) Serum ammonia levels can be of assistance in determining the extent of liver failure or gastrointestinal bleeding among other conditions. It is not the most helpful test in following up an elevated alkaline phosphatase. (Choice D) Prothrombin time is of assistance in assessing coagulative ability and liver functionality, especially in the acute setting. It is not the most helpful test in following up an elevated alkaline phosphatase. (Choice F) Aspartate aminotransferase is of assistance in assessing and monitoring hepatic damage. It is not the most helpful test in following up an elevated alkaline phosphatase. Educational Objective: A moderately elevated alkaline phosphatase of unclear etiology should be followed up with y-glutamyl transpeptidase.

283

USMLE WORLD STEP 1

PATHOPHYSIOLOGY

Q NO 78: A 56-year-old female presents to your office for routine check-up. She has no present complaints. Her past medical history is significant for a vertebral compression fracture experienced after a minor trauma one year ago. She has taken hormone replacement therapy since then. Her laboratory tests reveal normal serum TSH levels. Which of the following is most likely to be true in this patient? Increased total T4 pool B. Increased free T3 level C. Overactive iodine uptake by the thyroid D. Decreased conversion of T4 to T3 E. Occult sympathetic hyperactivity

A.

Explanation: This patient has osteoporosis (hence the fragility vertebral fracture) which is being treated with hormone replacement therapy containing estrogen and progesterone. Estrogen has an effect on thyroid hormones, which is detailed in the following paragraph—ultimately, estrogen causes an increase in total 14, but thyroid function remains normal. Similar changes are seen in the hyper-estrogenic state of pregnancy. More than 99% of circulating thyroid hormones are bound to plasma proteins. The main protein responsible for binding circulating thyroid hormone is thyroid binding globulin (TBG). TBG levels increase with estrogen use because the catabolism of TBG decreases when estrogen is present. An increase in TBG levels leads to an increase in total T4 (bound T4 plus free T4) and total T3. However the level of free thyroid hormones remains normal, so patients remain euthyroid and have normal ISH levels. (Choices B and E) As described above, levels of free thyroid hormones, including free T3, do not change with estrogen. Since the level of free thyroid hormones remains the same, patients in hyper-estrogenic states (pregnancy and oral contraceptive pill users) do not experience thyroid symptoms. Occult sympathetic activity would be found in a hyperthyroid patient. (Choices C and D) An increase in TBG levels neither leads to increased iodine uptake by thyroid gland nor to decreased peripheral conversion of T4 to T3. Medications that can decrease the peripheral conversion of T4 to T3 are propylthiouracil, glucocorticoids, amiodarone, iopanoic acid, and nonselective beta-blockers. Educational Objective: Increase in levels of thyroid binding globulin lead to increase in circulating total T4 and total T3. However the level of free thyroid hormone is normal. Increase in TBG is typically seen in pregnancy with use of oral contraceptives or with hormone replacement therapy.

284

USMLE WORLD STEP 1

PATHOPHYSIOLOGY

Q NO 79: A 10-year-old male is being evaluated for abnormal growth. He is in the 99th percentile for height and 90th percentile for weight. He has long and moist extremities. Both parents are proportionate and of average height. Serum TSH is within normal limits. Which of the following is the most likely cause of his condition? A. Increased release of insulin from pancreas B. Increased release of IGF-1 from liver C. Increased release of IGF-1 from hypothalamus D. Defective growth hormone receptors E. Excessive free thyroxine (T4) levels Explanation: Growth hormone is a 191-amino-acid hormone secreted from the anterior pituitary. Growth hormone increases linear growth—growth of the long bones— through increased production of insulin like growth factor-1 (IGF-1) from the liver. IGF-1 causes bone growth by stimulating the proliferation of cartilage in bone cells. Alternatively, if there is excess growth hormone before closure of the epiphysis, it is called “gigantism” a condition characterized by a tall stature and long limbs; after closure of the epiphysis, the condition is called “acromegaly.” Although gigantism and acromegaly are caused by the same excess hormone, the symptoms not alike. This patient is young enough that we can be sure that his epiphyses are not yet closed. It is unlikely that this patient is “just a tall kid” because his parents are of average height and he has long limbs; you should suspect gigantism. (Choice A) The main action of insulin is reduction of blood sugar levels, although it is true that insulin secretion also causes some ICE-i production from the liver. If insulin was being hyper-expressed (in a non-insulin resistance state), the patient would complain of symptoms of hypoglycemia—weakness, headache, nervousness, and/or syncope that resolve with food ingestion. (Choice C) Although growth hormone does affect synthesis of ICE-1 from the liver, it does not affect synthesis of ICE-1 from hypothalamus. ICE-1 that originates from the hypothalamus is believed to regulate central nervous activity and not to contribute significantly to a person’s linear growth. (Choice D) Defective growth hormone receptors will lead to decrease in linear growth and is called Laron dwarfism. In patients with defective receptors, serum levels of growth hormone will be high with low levels of circulating ICE-1. Growth hormone acts by binding to the cell surface receptor, which stimulates intracellular secondary pathway. The secondary pathways involved in growth hormone signaling are Janus kinase and STAT 5 (JAR-STAT) pathway. Growth hormone releasing hormone stimulates the release of growth hormone; whereas, somatostatin inhibits growth hormone’s release. (Choice E) Excessive circulating thyroid levels will suppress the TSH secretion from the pituitary. A slight increase in serum free T4 levels will have a marked effect on suppression of TSH release. The patient does not have increase free T4 levels because his TSH is within normal limits. Educational Objective: 1. Growth hormone increases linear growth by stimulating the production of IGF-1 from the liver. 2. Defective growth hormone receptors will lead to decrease in linear growth and is called Laron dwarfism. It is characterized by high serum levels of growth hormone in the presence of low IGF-1 levels.

285

USMLE WORLD STEP 1

PATHOPHYSIOLOGY

Q NO 80: A 35-year-old alcoholic is hospitalized with severe abdominal pain and vomiting. The patient’s condition fails to improve after initial treatment. Laparotomy reveals a swollen and partially necrotic pancreas. Activation of which of the following enzymes is most likely responsible for this patient’s condition? Chymotrypsin B. Trypsin C. Phospholipase D. Elastase E. Lipase F. Amylase

A.

Explanation: Pancreatic enzymes are secreted by pancreatic acinar cells as zymogens, inactive enzyme precursors. Zymogen granules are excreted from the apical surface of acinar cells by exocytosis. The zymogens then enter the pancreatic ductal system and drain through the ampulla of Vater into the descending part of the duodenum. There the duodenal brush border enzyme enterokinase turns trypsinogen into trypsin, its active form. Once a small quantity of trypsin is produced, it activates most of the other proteolytic enzymes, such as chymotrypsin, elastase, carboxypeptidase and others. It addition, trypsin activates trypsinogen to produce more trypsin. This ability to self-activate allows trypsin to maintain a self-supporting cycle of proteolytic enzyme activation in the duodenum. In acute pancreatitis, inflammation and ischemia of pancreatic tissue lead to acinar cell damage. This triggers abnormal activation of trypsin inside the acinar cells. Trypsin then activates the other proteolytic enzymes and starts a self-sustaining cycle of digestion of pancreatic tissues. Pancreatic autodigestion (autolysis) is the central event in the pathogenesis of acute necrotizing pancreatitis. (Choices A, C and D) Chymotrypsin, phospholipase A2, and elastase are secreted by the pancreas as inactive precursors (chymotrypsinogen, pyrophosphokinase A2, and proelastase) which are subsequently activated by trypsin in the duodenal lumen. (Choice E) Lipase hydrolase triglycerides into fatty acids and glycerol. It is synthesized in an active form and does not require activation by trypsin. (Choice F) Amylase hydrolyzes starch to produce maltose (a glucose-glucose disaccharide) trisaccharide maltotriose, and limit dextrins. It does not require activation by trypsin. Educational Objective: The abnormal activation of trypsin within the pancreas is a central event in the pathogenesis of acute necrotizing pancreatitis. All proteolytic pancreatic enzymes are converted into their active forms by trypsin. Intracellular pancreatic activation of trypsin leads to activation of other proteolytic enzymes and pancreatic autodigestion.

286

USMLE WORLD STEP 1

PATHOPHYSIOLOGY

Q NO 81: A 74-year-old Caucasian female presents to the physician’s office for evaluation of choking spells, dysphagia, and coughing. She suffers from recurrent pneumonia and has a long hi stow of hypertension, but no other medical problems. Barium swallow is shown below. Which of the following is the most likely cause of her symptoms?

A. Degenerative changes of the myenteric plexus B. Cricopharyngeal motor dysfunction C. Scarring and traction of the esophagus D. Retention cyst due to duct obstruction E. Increased intraluminal pressure in the stomach

Explanation: Deglutition (swallowing) is a complex process that includes three phases. The oral phase is voluntary: the food bolus is collected at the back of the mouth and lifted upwards to the posterior wall of the pharynx. This initiates the pharyngeal phase, which consists of involuntary pharyngeal muscle contractions that propel the food bolus to the esophagus. During the esophageal phase, food enters the esophagus and stretches the walls. Peristalsis begins just above the site of distention and moves the food downward. Relaxation of the lower esophageal sphincter (LES) follows, allowing the food bolus to enter the stomach. Cricopharyngeal muscle dysfunction occurs due to diminished relaxation of pharyngeal muscles during swallowing. More force is subsequently required to move the food bolus downward. More intense contractions of the pharyngeal muscles increase the oropharyngeal intraluminal pressure. With time, the pharyngeal mucosa will herniate through muscle fibers in the zone of weakness (posterior hypopharynx), forming a Zenker diverticulum. Remember that when a diverticulum consists only of mucosa, itis a false or pulsion, diverticulum. A traction diverticulum, alternatively, consists of all layers of the organ wall. Cricopharyngeal dysfunction causes symptoms of “high dysphagia:” difficulty in swallowing felt “at the throat” coughing choking and sometimes even nasal regurgitation. When a Zenker diverticulum forms it can cause food retention with regurgitation occurring days later. Food aspiration may lead to pneumonia, as happened in this patient. The diverticulum can be palpated as a lateral neck mass. (Choice A) Degenerative changes of the myenteric plexus result in achalasia. (Choice C) Scarring and traction of the esophagus result in true diverticula. True diverticuli are usually seen at the mid-portion of the esophagus and result from mediastinal lymphadenitis (tuberculosis fungal infections). The above image shows a diverticulum at the upper part of the esophagus.

287

USMLE WORLD STEP 1

PATHOPHYSIOLOGY

(Choice D) Retention cysts are formed if the duct of a gland is obstructed trapping secretions. Cricopharyngeal muscle dysfunction is not associated with the formation of retention cysts. (Choice E) Mallory-Weiss syndrome describes mucosal tears that occur around the esophagogastric squamocolumnar junction. These tears are caused by the increased intraluminal pressure in the stomach that occurs during retching or vomiting. Educational Objective: Cricopharyngeal muscle dysfunction is a condition caused by diminished relaxation of pharyngeal muscles during swallowing. The subsequently increased intraluminal pressure in the oropharynx causes the mucosa to herniate through the wall at a point of muscle weakness, forming a Zenker diverticulum. Clinically, patients (elderly) present with oropharyngeal dysphagia, coughing, choking, and recurrent aspiration.

288

USMLE WORLD STEP 1

PATHOPHYSIOLOGY

Q NO 82: A 21-year-old male presents to his physician after noticing that his urine had a “frothy” appearance. He also complains of easy fatigability and anorexia. His past medical history is significant only for an upper respiratory infection several weeks ago. Physical examination reveals symmetric pitting edema of the ankles. Which of the following is most likely decreased in this patient? A. Capillary hydrostatic pressure B. Interstitial fluid pressure C. Plasma oncotic pressure D. Tissue lymphatic drainage E. Circulating aldosterone level Explanation: Frothy, foamy urine may be caused by proteinuria or bile salts in the urine. This patient’s history of a recent upper respiratory infection and ankle edema on physical exam suggest a diagnosis of nephrotic syndrome with associated low serum albumin. Hypoalbuminemia lowers the plasma oncotic pressure and causes interstitial edema formation due to net plasma filtration. Minimal change disease (MCD) is the most common cause of nephrosis in children, and can occur in adults as well. (Choice A) A decrease in capillary hydrostatic pressure would tend to decrease net plasma filtration and interstitial edema formation. (Choice B) This patient’s ankle edema is the result of a transudate of plasma into the interstitial tissues of the ankle. We would therefore expect an increase in the steady state interstitial fluid pressure in the ankles. (Choice D) While a primary decrease in lymphatic drainage can cause interstitial edema, the rate of lymphatic drainage would be increased in this particular patient because of the accumulation of ankle interstitial fluid. (Choice E) In nephrotic syndrome, the plasma oncotic pressure is decreased, which causes net plasma filtration into the interstitium, thus decreasing the effective circulating intravascular volume. This reduction of the intravascular volume stimulates a compensatory increase in the activity of the renin-angiotensinaldosterone system. Patients with nephrotic syndrome tend to have elevated circulating aldosterone levels. Educational Objective: Frothy or foamy urine maybe caused by proteinuria. Heavy proteinuria, as in nephrotic syndrome, can cause regional or generalized interstitial edema because the decrease in serum albumin and total protein concentrations lowers the plasma oncotic pressure and increases net plasma filtration in capillary beds.

289

USMLE WORLD STEP 1

PATHOPHYSIOLOGY

Q NO 83: A 63-year-old Caucasian male presents to the emergency department with severe dyspnea, orthopnea and fatigue. He suffered a myocardial infarction six months ago, and has not been compliant with his medications since that time. On exam, his blood pressure is 170/100 mmHg and his heart rate is 100 beats per minute. Auscultation reveals crackles at the lung bases, an S3 gallop, and a Il/VI holosystolic murmur over the apex. After initial treatment with diuretics and vasodilators the patient’s condition improves significantly. The next morning, there are no appreciable gallops or murmurs on cardiac exam. The murmur heard at the time of presentation is most likely explained by: Thickened and deformed mitral valve cusps B. Heavily calcified mitral annulus C. Increased flow rate through the aortic valve D. Ruptured chorda tendineae E. Functional mitral regurgitation

A.

Explanation: This patient’s symptoms of dyspnea and orthopnea together with bibasilar crackles are consistent with high pulmonary venous pressure and pulmonary edema in the dependent lung. The holosystolic murmur heard over the cardiac apex is suggestive of mitral regurgitation (MR). An 33 gallop reflects an increased left ventricular filling rate during mid diastole, and can be heard as a consequence of MR. Treatment with a diuretic tends to reduce left ventricular (LV) preload and therefore decreases the LV end diastolic volume (EDV). Since the patient’s murmur and gallop disappeared following the reduction of LVEDVI his MR was most likely functional— that is, due to transient hemodynamic factors causing LV dilatation and/or papillary muscle ischemia rather than due to a fixed mitral valve lesion. Acute LV dilatation can sufficiently separate otherwise normal mitral valve leaflets to permit functional regurgitation. The most common anatomical abnormality producing mitral regurgitation is myxomatous degeneration (mitral valve prolapse). Afterload reduction with a vasodilator decreases the average intraventricular systolic pressure required to generate a given stroke volume and would tend to reduce MR due to any cause. (Choice A) Thickened and deformed mitral valve cusps are fixed anatomical lesions that typically result from chronic rheumatic heart disease. For this reason, these lesions are usually only found in older individuals (who may not have had access to antibiotics while young). Mitral stenosis is the usual result. (Choice C) Mitral annular calcification, consisting of degenerative calcific deposits in the fibrous ring of the mitral valve, generally does not impair valvular function. There is associated regurgitation or stenosis only in rare instances. Mitral annular calcification is most common in women older than 60, individuals with a history of myxomatous degeneration of the mitral valve, and individuals with chronically elevated left ventricular (LV) pressures. Furthermore, whereas the patient in the vignette had mitral regurgitation that was eliminated by a reduction in LV size (which also decreases the mitral valve radius), the radius is fixed in patients with a calcified mitral valve annulus. (Choice D) An increased rate of flow through the aortic valve could produce a functional murmur of aortic stenosis, which would be a systolic ejection type murmur heard best over the aortic area (right upper sternal border). (Choice E) Chordae tendineae rupture producing severe mitral regurgitation (MR) is a complication of bacterial endocarditis, and less frequently of connective tissue diseases or acute myocardial infarction. When papillary muscle or chordae rupture

290

USMLE WORLD STEP 1

PATHOPHYSIOLOGY

occurs in association with myocardial infarction, it is usually an early complication of the Ml (i.e. occurring within 10 days). The MR of the patient in the vignette appears to have been precipitated by medication non-compliance which caused an acute hemodynamic change. Thus endocarditis and connective tissue disease are more remote possibilities. Additionally, the murmur of MR due to chordae rupture may not be completely eliminated by pharmacological manipulations. Chordae rupture results in fixed anatomic (versus reversible functional) MR. Educational Objective: Acute hemodynamic changes can produce functional heart murmurs in the absence of any fixed valve lesion. Dilatation of the left ventricle in response to increased preload can result in functional mitral regurgitation which can be eliminated by preload reduction and reduced by afterload reduction.

291

USMLE WORLD STEP 1

PATHOPHYSIOLOGY

Q NO 84: A 34-year-old immigrant from South Asia presents to your office complaining of heart palpitations that are particularly prominent at night. He also notes that with moderate exertion, he experiences head “pounding” accompanied by involuntary head bobbing. He remembers being diagnosed with a heart murmur years before, but he cannot recall the type and has never received any treatment. Based on this patient’s history, you suspect: Restricted left ventricular filling B. Impaired left ventricular contractility C. Left ventricular outflow obstruction D. Systolic-diastolic hypertension E. Widening of the pulse pressure

A.

Explanation: This patient complains of nocturnal palpitations and head pounding with exertion. Palpitations may result from forceful ventricular contractions ejecting large stroke volumes, and head pounding can be due to unusually high amplitude pulsations of the intracranial arteries with each heartbeat. In voluntary head bobbing can be a sign of a widened pulse pressure (recall that pulse pressure = peak systolic arterial pressure — end diastolic arterial pressure). The most likely cause of a repetitive, widened pulse pressure together with unusually large LV stroke volumes and a heart murmur is aortic regurgitation (AR). (Choice A) Restriction of left ventricular (LV) filling would result in a reduced LV end diastolic volume (reduced preload). At a given level of contractility, this would cause a reduction in stroke volume. Lower stroke volumes result in lower pulse pressures, whereas this patient’s symptoms and signs suggest a high pulse pressure. (Choice B) Impaired left ventricular contractility would cause a reduction in stroke volume for a given preload, resulting in lower pulse pressures. (Choice C) Left ventricular outflow tract obstruction, as can result from aortic stenosis or hypertrophic cardiomyopathy, could cause a murmur but would tend to reduce stroke volume and thus pulse pressure. (Choice D) Combined systolic and diastolic hypertension is not necessarily accompanied by an abnormally large pulse pressure or murmur. Educational Objective: The abnormally large (wide) pulse pressure caused by aortic regurgitation (AR) is responsible for many of the symptoms and signs of AR.

292

USMLE WORLD STEP 1

PATHOPHYSIOLOGY

Q NO 85: In an animal experiment, the left anterior descending (LAD) coronary artery is ligated (time 0) and changes in substance levels within the affected myocardial cells are recorded. At which point on the graph does the affected myocardium stop contracting?

A. B. C. D. E.

A B C D E

Explanation: The main early biochemical consequence of total myocardial ischemia is cessation of aerobic glycolysis and initiation of anaerobic glycolysis. This transition occurs within seconds, and results in inadequate production of high-energy phosphates (e.g. ATPI creatine phosphate) to sustain cardiac muscle contraction. Loss of contractility and lactate accumulation begin within 60 seconds of total ischemia. (Choice B) After 5 minutes of total myocardial ischemia, ATP levels within the affected cardiac myocytes have fallen to JS% of normal, and lactate levels have risen significantly. (Choice C) After 10 minutes of total myocardial ischemia, intracellular ATP levels in the affected myocytes have been reduced to 50% of normal, and lactate levels are markedly elevated. (Choice D) After 20 minutes of total myocardial ischemia, intracellular ATP levels in the affected myocytes have been reduced to about 25% of normal and lactate levels are high. (Choice E) After 30 minutes of total myocardial ischemia, intracellular ATP levels in the affected myocytes have been reduced to about 15% of normal and lactate levels are very high. When total ischemia has persisted for 30 minutes, ischemic injury becomes irreversible. In contrast, when the ischemia lasts less than 30 minutes myocardial stunning (reversible loss of contractile function) occurs. Educational Objective: Loss of cardiac myocyte contractile function occurs within 60 seconds of the onset of total ischemia, due to immediate cessation of aerobic glycolysis and initiation of anaerobic glycolysis, with a fall in high-energy phosphates such as ATP and creatine phosphate. After 30 minutes of total ischemia, when ATP levels in affected myocytes have dropped to 15% of normal and lactate levels are very high, irreversible ischemic injury is likely.

293

USMLE WORLD STEP 1

PATHOPHYSIOLOGY

Q NO 86: A 23-year-old Caucasian male with periodic abdominal pain is diagnosed with a duodenal mucosal defect on upper gastrointestinal contrast X-ray. Duodenal mucosal samples taken during endoscopy demonstrate H. pylori colonization. Which of the following accounts for bacterial colonization in the duodenum of this patient? A. Increased bicarbonate secretion B. past dumping of gastric content C. Areas of mucosal necrosis D. Mucosal metaplasia E. Mucosal inflammation Explanation: Most cases of duodenal and gastric ulcers are associated with Helicobacter pylori, a gram-negative microaerophilic rod that resides between the protective mucus layer and gastric epithelium in colonized persons. The following bacterial virulence factors are implicated in the development of chronic gastritis and peptic ulcer disease (PUD): 1. H. pylori produce urease. This enzyme catalyzes the synthesis of ammonia from urea, causing an increase in pH. 2. H. pylori possess adhesins. These substances enable the bacteria ‘to adhere” to gastric epithelium. 3. H. pylori secrete proteases and phospholipases. These enzymes break down gastric mucus. 4. A number of H. pylori surface factors attract both neutrophils and monocytes, which contribute to epithelial injury. 5. Some of the enzymes of H. pylori irritate the gastric mucosa, causing inflammation. All of the above virulence factors allow H. pylori to thrive on the surface of gastric epithelium; however, H. pylori cannot live on intestinal epithelium. Therefore, the presence of H. pylori in the duodenum indicates that gastric-type metaplasia of the intestine is present. It is thought that H. pylori colonization of the stomach increases gastric acid production, which leads to acidic injury of duodenal mucosa, inducing gastric metaplasia there. Then, these metaplastic “shelters” for H. pylon! cause inflammation in surrounding duodenal epithelium. Alternatively, some researchers feel that the gastric metaplasia may be pre-existent in some patients. (Choice A) H. pylori infection is associated with decreased bicarbonate secretion in the duodenum, which weakens the defense barrier of the duodenal mucosa to infection. (Choice B) Fast dumping of acidic gastric contents into the duodenum is sometimes noted in patients with duodenal ulcers. However, it is not considered a factor of H. pylori survival in the duodenum. (Choices C and E) Mucosal inflammation and subsequent mucosal necrosis can result from the presence of H. pylori in the gastric or duodenal lumen. These mucosal changes are the result of, not the facilitator for, survival in the duodenum. Educational Objective: H. pylori is well-adapted to live in the gastric lumen. This pathogen doesn’t colonize normal duodenal mucosa, only gastric-type mucosa. The full etiology of duodenal ulcers is still unclear. Areas of gastric metaplasia may be preexistent in the duodenum or they may develop secondary to acid injury caused by H. pylori colonization of the stomach.

294

USMLE WORLD STEP 1

PATHOPHYSIOLOGY

Q NO 87: A 55-year-old Caucasian female presents to your office complaining of weight gain and easy fatigability. She takes a multivitamin and some over-thecounter medications for constipation. Her blood pressure is 140/96 mmHg, and heart rate is 55/mm. Physical examination reveals a delayed relaxation of knee and ankle reflexes. Skin is very dry and thick. Which of the following is the single best screening test to confirm the diagnosis in this patient?

A. Serum T3 and T4 levels B. Serum TSH level C. 24-hour urinary cortisol excretion D. Dexamethasone suppression test E. Serum prolactin level

Explanation: The features described above—fatigue, weight gain, constipation, slowed relaxation of deep tendon reflexes, and dry, coarse skin—are strongly suggestive of primary hypothyroidism. The pituitary gland secretes thyroid stimulating hormone (TSH) or thyrotrophin which increases the synthesis and release of thyroid hormones from the thyroid gland. In turn, the pituitary itself is stimulated by the hypothalamus, which secretes thyrotrophin-releasing hormone (TRH). Excessive thyroid hormone release causes inhibition of TSH and TRH production and release in a classic negative feedback mechanism. Small changes in thyroid hormone levels lead to marked changes in serum TSH level. In hypothyroidism, the TSH rise occurs well before a low thyroid hormone level is seen. Some changes also occur at the level of hypothalamus but TRH is difficult to measure. Thus, serum TSH is the most sensitive marker for diagnosis of hypothyroidism. The main caveat for using TSH in the diagnosis of hypothyroidism is that itis not elevated in patients with hypothyroidism due to TSH deficiency (central hypothyroidism). Nonetheless central hypothyroidism is relatively uncommon and there are usually other clinical features that suggest its presence. (Choice A) T4 levels can be low but within normal limits in early mild primary hypothyroidism. Since TSH levels rise beforeT4 levels are low TSH is a more sensitive test. Serum T3 is the last to decline in most patients with hypothyroidism. (Choices C and D) Twenty-four hour urinary cortisol and the dexamethasone suppression test are screening tests for Cushing syndrome. The patient described above does not have any features of Cushing syndrome which include central

295

USMLE WORLD STEP 1

PATHOPHYSIOLOGY

obesity, skin striae rounded facies deposition of supraclavicular fat and proximal weakness. (Choice E) Hyperprolactinemia characteristically leads to amenorrhea and galactorrhea in females. Increased serum prolactin can occur in hypothyroidism due to the stimulatory effect of elevated hypothalamic TRH, which then stimulates the pituitary causing excess prolactin production and release. Educational Objective: Serum TSH level is the single most important screening test in diagnosing primary hypothyroidism. Although TSH is not elevated in patients with hypothyroidism in central hypothyroidism central hypothyroidism is uncommon.

296

USMLE WORLD STEP 1

PATHOPHYSIOLOGY

Q NO 88: A group of investigators studies the process of bone remodeling in response to steroid hormones. In animal experiments they note that the castration of young females (i.e. removal of the ovaries) leads to overexpression of RANK receptors on the surface of some bone cells. Which of the following is the most likely consequence of the response described by the investigators? Increased bone resorption B. Decreased osteoid formation C. Decreased mineralization D. Osteocyte apoptosis E. Decreased blood flow to the bone

A.

Explanation: Osteoclasts are derived from the hemopoietic monocyte phagocytic cell lineage. The differentiation of osteoclasts is mainly governed by the RANK-ligand and by monocyte colony-stimulating factor, both of which are produced by osteoblasts. Interaction of RANK-ligand and monocyte colony-stimulating factor on receptors present in osteoclastic precursors stimulates the development of mature, multi nucleated osteoclasts. The interaction of RANK with its ligand is decreased by another protein secreted by the osteoblast—this protein is called osteoprotegerin (OPG) and it acts as a decoy receptor. Bone turnover is regulated by the ratio of RANK-ligand to OPO. Bone turnover increases when RANK-L is high and OPO is low. Similarly, the over expression of RANK receptors on osteoclasts also increases bone turnover. Parathyroid hormone (PTH) stimulates osteoclasts in an unexpected way. Osteoclasts do not have PTH receptors—PTH acts on osteoblasts, instead. PTH stimulates the secretion of monocyte colony-stimulation factor and RANK-ligand by osteoblasts, thus stimulating osteoclastic precursor to become mature osteoclasts. (Choices B and C) RANK receptors are not present on osteoblasts, so an increased expression of the RANK receptor will have no effect on osteoid formation. In normal subjects, the resorption and formation of bone is tightly coupled. In patients who are estrogen deficient, resorption is higher than formation; the net effect is bone loss. (Choice D) Osteocytes are the precursors of osteoblasts. RANK receptors are not expressed on osteocytes. (Choice E) Lower estrogen levels lead to increased vascular endothelial growth factor (VEGE) production locally within the bone. Vascular endothelial growth factor augments new blood vessel formation and increases blood flow. Mice that have been spayed will have an increase in bone vascularity due to VEGE, not decreased blood flow to the bone. Educational Objective: The RANK receptor/RANK-ligand interaction is essential for the formation and differentiation of osteoclasts. The overexpression of RANK receptors in hypoestrogenic states causes increased bone resorption due to increased osteoclastic activity. In short, low estrogen means a lower bone mass.

297

USMLE WORLD STEP 1

PATHOPHYSIOLOGY

Q NO 89: A 27-year-old Caucasian male presents to your office complaining of repetitive episodes of muscle weakness. He has no significant past medical history. His blood pressure is 190/110 mmHg while supine and 195/110 mmHg while standing. His heart rate is 70 beats per minute while supine and 72 beats per minute while standing. Laboratory evaluation reveals very low plasma renin activity. Overactivity of which of the following structures is most likely responsible for this patient’s symptoms? Chromatin cells of the adrenals B. Extra-adrenal paraganglion cells C. Zona glomerulosa of the adrenals D. Zona fasciculata of the adrenals E. Zona reticulata of the adrenals F. Juxtaglomerular cells of the kidney

A.

Explanation: The adrenal glands are situated above the kidneys and consist of an outer cortex and an inner medulla. The adrenal cortex develops and functions independently from the adrenal medulla. The mature adrenal cortex consists of three distinct zones: the outer zona glomerulosa the middle zona fasciculata, and the inner zona reticularis (Remember GFR). The zona glomerulosa synthesizes mineralocorticoids, the zona fasciculata predominantly produces cortisol, and the zona reticularis predominantly produces androgens. Secretion of aldosterone from the zona glomerulosa is regulated by the reninangiotensin system and potassium. Because the zona glomerulosa lacks 17-ahydroxqlase, it cannot synthesize cortisol or androgens. ACTH from the anterior pituitary increases the secretion of cortisol and adrenal androgens from the zona fasciculata and zona reticularis, respectively but there is no substantial effect of ACTH on aldosterone secretion. Aldosterone’s main effect is to stimulate sodium absorption and potassium and hydrogen ion excretion at the distal renal tubule. Thus overproduction of aldosterone by tumors or hyperplastic zona glomerulosa cells can result in sodium retention hypertension hypokalemia, and metabolic alkalosis. Despite this increase in sodium absorption, however hypernatremia is rarely observed with mineralocorticoid excess, due to the phenomenon of “aldosterone escape.” The high aldosterone levels cause increased renal sodium and water absorption, thus increasing renal blood flow and GFRI which in turn increase the rate of sodium excretion from the renal tubules. Furthermore, the increase in intravascular volume stimulates the release of atrial natriuretic peptide, which causes natriuresis. Thus, hypertension, hypokalemia and metabolic alkalosis, without marked hypernatremia, characterize mineralocorticoid excess. Renin levels are typically very low, due to the hypervolemia. The hypokalemia of hyperaldosteronism can cause profound muscle weakness. (Choice A and B) Pheochromocytoma is a neuroendocrine tumor of the chromaffin cells of the sympathetic nervous system (90% arising from the adrenal medulla. 10% arising extra-adrenally along the sympathetic chain). Hypokalemia is not a feature of pheochromocytoma, and renin levels may actually be increased secondary to decreased intravascular volume. (Choice D and E) Hypersecretion of the zona fasciculata or zona reticularis (secondary to hyperplasia or neoplasia) would lead to Cushing syndrome or adrenal hyperandrogenism, respectively. (Choice F) When there is increased renin production by the juxtaglomerular apparatus aldosterone levels increase. and hypertension can result. This typically

298

USMLE WORLD STEP 1

PATHOPHYSIOLOGY

occurs in the setting of renal artery stenosis, when decreased renal blood flow stimulates renin release from the juxtaglomerular apparatus. Increased renin levels cause increased formation of angiotensin I, which is subsequently converted to angiotensin II. Angiotensin II directly increases blood pressure via vasoconstriction and indirectly increases blood pressure by stimulating aldosterone release. Secondary hyperaldosteronism resulting from renal artery stenosis causes hypokalemia and hypertension. Thus, patients with hypertension secondary to renal artery stenosis have high renin and aldosterone levels. (Primary aldosteronism results from adrenocortical adenomas or hyperplasia. Circulating renin levels are very low in this condition.) Educational Objective: The adrenal cortex consists of three distinct zones: the outer zona glomerulosa the middle zona fasciculata, and the inner zona reticularis (remember GFR). The zona glomerulosa synthesizes mineralocorticoids (e.g. aldosterone), the zona fasciculata predominantly produces cortisol, and the zona reticularis predominantly produces androgens.

299

USMLE WORLD STEP 1

PATHOPHYSIOLOGY

Q NO 90: An infant born to a 22-year-old female has a small phallus and hypospadias. His testes are well developed but reside in the inguinal area. His blood pressure and serum testosterone levels are normal. His karyotype is 46XY. Which of the following is most likely deficient in this patient? hydroxqlase B. Aromatase C. 17-hydroxylase D. 5a-reductase E. DHEA sulfatase

A. 21-

Explanation: 5a-reductase converts testosterone into dihydrotestosterone (DHT). Testosterone is responsible for development of the internal male genitalia during embryogenesis, spermatogenesis, and male sexual differentiation at puberty (such as development of muscle mass and libido). DHT mediates development of the external genitalia (penis and scrotum) in the embryo growth of the prostate, facial hair and temporal recession of the hairline. DHT also amplifies the effects of testosterone due to a much higher affinity for the testosterone receptor. There are two types of 5α-reductase. Type 1 is present in postpubescent skin, while type 2 is found in the genitals. Deficiency of 5α-reductase type 2 results in diminished conversion of testosterone to DHT in the tissues. In the male fetus with this genetic defect, the internal genitalia develop normally under the influence of testosterone, but the external genitalia do not develop properly due to the lack of DHT resulting in male pseudohermaphroditism. The genitalia at birth can range from a small phallus with hypospadias as described in the question to ambiguous or female-type at birth. Such children may be raised as females until they reach puberty when high levels of testosterone and the action of Type 1 5a-reductase results in masculinization evidenced by male-pattern muscle mass, voice deepening, penile and scrotal growth, and testicular descent. (Choice A) 21-hydroxylase converts 17-hydroxyprogesterone into 11-deoxcortisol, and progesterone into 11- deoxycorticosterone. Corticosteroid precursors are shunted toward androgen production resulting in virilization of the female fetus and salt wasting. (Choice B) Aromatase catalyzes the conversion of androgens to estrogens in the gonads and peripheral tissues. Aromatase deficiency presents with virilization of female infants; male patients are not phenotypically affected. (Choice C) Deficiency of 17-hydroxylase results in decreased secretion of cortisol and sex steroids and an increased level of mineralocorticoids. Clinically it manifests with sodium retention leading to hypertension and under virilization of male infants (female-type genitalia). (Choice E) DHEA sulfate is a weak androgen produced by the adrenal cortex. Deficiency of DHEA sulfatase does not result in under virilization of the male fetus. Educational Objective: 5α-reductase converts testosterone to dihydrotestosterone (DHT). DHT mediates development of the external genitalia in the male fetus. Male neonates with 5areductase deficiency are born with feminized external genitalia that typically masculinize at puberty. Small phallus and hypospadias are commonly found.

300

USMLE WORLD STEP 1

PATHOPHYSIOLOGY

Q NO 91: Coronarii angiography demonstrates several atherosclerotic lesions in an asymptomatic 35-year-old Caucasian male. The lesions do not cause significant coronary flow obstruction. High intraplaque activity of which enzyme would predispose the patient to myocardial infarction? A. Prolyl hydroxylase B. Lysyl oxidase C. Procollagen peptidases D. Metalloproteinases E. Heparinase Explanation: Plaque rupture can produce an acute coronary syndrome via superimposed thrombosis and/or thromboembolism. The likelihood of plaque rupture or other acute plaque change has more to do with plaque stability than plaque size. Plaque stability depends upon the mechanical strength of the overlying fibrous cap. A weak fibrous cap increases the probability of plaque rupture. During the chronic inflammatory progression of an atheroma, the fibrous cap is continually being remodeled. The balance of collagen synthesis and degradation determines the mechanical strength of the cap. Activated macrophages in the atheroma contribute to collagen degradation by secreting metalloproteinases. Thus, a high degree of ongoing intimal inflammation can destabilize the mechanical integrity of plaques through release of these metalloproteinases. High intraplaque activity of such enzymes predisposes the patient to plaque rupture and a consequent acute coronary syndrome, including myocardial infarction. (Choice A) The enzyme prolyl4-hydroxqlase is responsible for the hydroxylation of proline on procollagen chains, a required step in the formation of a stable collagen triple helix. Enhanced collagen deposition would strengthen the fibrous cap, thereby stabilizing the atheroma. (Choice B) Lysyl oxidase catalyzes the conversion of the amino terminus of lysine residues to an aldehyde in collagen fibers. Extracellular collagen fibers are strengthened by this mechanism. Thus, a high activity of lysyl oxidase would favor atheroma stabilization by strengthening collagen in the fibrous cap. (Choice C) Procollagen peptidase cleaves the terminal regions of procollagen molecules exocytose by fibroblasts or SMCI transforming the procollagen into insoluble tropocollagen. Tropocollagen then aggregates to form collagen fibrils. The activity of procollagen peptidase is deficient in Ehlers-Danlos syndrome, a condition characterized by disorganized collagen bundles. These patients have stretchable skin hypermobile joints short stature, and other connective tissue defects. (Choice E) Heparinase is not an endogenous enzyme. However, if it were, an increase in its activity could conceivably be prethrombotic or thrombogenic, theoretically predisposing a patient with coronary artery plaques to myocardial infarction. Educational Objective: The likelihood of acute plaque change (especially rupture) producing an acute coronary syndrome via superimposed thrombosis and/or thromboembolism is most related to plaque stability. Plaque stability depends significantly on mechanical strength of the fibrous cap. Inflammatory macrophages in the intima may reduce plaque stability by secreting metalloproteinases, which degrade collagen.

301

USMLE WORLD STEP 1

PATHOPHYSIOLOGY

Q NO 92: A 58-year-old Caucasian male is hospitalized by the sudden onset of chest pain. His blood pressure is 160/110 mmHg, and his heart rate is 90/mm. His BMI is 30.5 kg/rn2. A baseline ECG reveals non-specific ST segment and T wave changes, and serial troponin measurements are normal. His fasting plasma glucose level is found to be 160 mg/dL, although he has never been diagnosed with diabetes mellitus. Which of the following would correlate most with the resistance of tissues to the action of insulin in this patient? A. Arm circumference B. Triceps area skin thickness C. Waist-to-hip ratio D. Increase in liver glycogen F. Urinary ketone excretion Explanation: The patient described in this vignette has hypertension obesity and possible type 2 diabetes mellitus. The pathophysiology of type 2 diabetes mellitus involves insulin resistance along with defective insulin secretion from pancreatic beta cells. Although still controversial, many researchers believe that insulin resistance is the main defect in type 2 diabetes mellitus. Insulin resistance is a heterogenous disorder caused by number of genetic and environmental factors. Genetic defects include receptor and post-receptor mutations that result in faulty insulin signaling. Environmental factors that increase insulin resistance include lack of physical activity and obesity. Increased BMI is very commonly associated with insulin resistance and type 2 diabetes mellitus. A high BMI typically connotes a higher body fat content, although a very muscular person (like a bodybuilder) might have a high BMI, also. There are two different types of fat in the body: visceral fat and subcutaneous fat. In general, the visceral deposition of fat (fat surrounding internal organs) has a much stronger correlation with insulin resistance than does subcutaneous fat. Measuring of the waist-to-hip ratio (VVHR) indirectly measures the visceral fat to subcutaneous fat as the abdomen contains mainly viscera and hips have only subcutaneous fat. A high waist hip ratio is associated with insulin resistance, metabolic syndrome and type 2 diabetes mellitus. The metabolic syndrome is a group of risk factors that include hypertension, abdominal obesity, atherogenic dyslipidemia, insulin resistance and other factors. (Choices A and B) Measuring the whole arm circumference or triceps skin fold thickness correlates poorly with insulin resistance and type 2 diabetes mellitus. These parameters indirectly measure muscle mass and/or subcutaneous fat deposition. (Choice D) An increase in liver fat suggests insulin resistance; whereas, an increase in glycogen deposition suggests enhanced insulin action (after glycogen storage diseases have been excluded). Insulin increases glycogen synthesis in the liver and in skeletal muscles. (Choice E) Increase in urine ketone production usually occurs with absolute insulin deficiency as seen in type 1 diabetes mellitus. Although type 2 diabetics are relatively deficient in insulin the absence is not complete. In fact patients with type 2 diabetes mellitus generally have high circulating insulin which suppresses ketogenesis. Educational Objective: Visceral obesity as measured by waist-to-hip ratio is an important determinant of insulin resistance.

302

USMLE WORLD STEP 1

PATHOPHYSIOLOGY

Q NO 93: A 30-year-old female has had diarrhea and weight loss for several months. She also has diffuse bone pain and weakness. Her diarrhea improves with a gluten-free diet. Which of the following is most likely seen on the laboratory evaluation of this patient? A. B. C. D. E.

Serum Ca increased decreased decreased decreased normal

Serum phosphorous decreased decreased increased increased increased

Serum PTH increased increased decreased increased increased

Explanation: This woman has celiac disease, which is characterized by an increased sensitivity to a group of dietary proteins present in wheat barley, and rye collectively known as “gluten.” In patients who suffer from celiac disease the small intestinal mucosa becomes inflamed and atrophic in response to gluten, which results in defective mucosal absorption of nutrients. Celiac disease can also cause vitamin D malabsorption, which causes rickets in children and osteomalacia in adults. In vitamin D deficiency, both calcium and phosphorus absorption from the gastrointestinal tract are markedly decreased. Low levels of 1, 25 dihydroxyvitamin D, and the ensuing hypocalcemia, cause an increase in parathyroid hormone levels (secondary hyperparathyroidism). In vitamin D deficiencies, the serum phosphorus is decreased because there is an increase in urinary excretion of phosphorus induced by high PTH levels and because phosphorous is not properly absorbed from the gastrointestinal tract. (Choice A) An elevated PTH, hypercalcemia, and hypophosphatemia are the values expected in patients with primary hyperparathyroidism. (Choice C) In patients with hypoparathyroidism, the decrease in serum PTH level is accompanied by a decrease in serum calcium and an increase in serum phosphorus. (Choices D and E) Either of these biochemical scenarios can occur in pseudohypoparathyroidism and chronic renal failure. Resistance to PTH causes pseudohypoparathyroidism. The patient will have biochemical features of hypoparathyroidism (normal to low calcium levels and elevated phosphorous levels) but will have elevated circulating levels of parathyroid hormone. In chronic renal failure, high PTH is accompanied by normal to low serum calcium and increase in serum phosphorus. Low serum calcium is due to the decreased production of 1,25dihydroxivitamin D from the damaged kidneys, resulting in decreased calcium absorption from the gastrointestinal tract. Chronic renal disease results in impaired phosphorus excretion leading to hyperphosphatemia. Educational Objective: The diarrhea of celiac disease can lead to vitamin D deficiency through malabsorption. Patients with a vitamin D deficiency have decreased serum phosphorus increased serum PTH (secondary hyperparathyroidism), and low serum calcium. They may also have symptoms such as bone pain and muscle weakness.

303

USMLE WORLD STEP 1

PATHOPHYSIOLOGY

Q NO 94: A one-month-old Caucasian male with a history of persistent jaundice experiences muscle rigidity, lethargy and seizures. Which of the following causes of hyperbilirubinemia is most likely to produce this patient’s neurological abnormalities? Absent liver conjugation enzymes B. Deficient bilirubin excretion into bile canaliculi C. Impaired canalicular bile transit D. Increased gut deconjugation of bilirubin E. Impaired gut reabsorption of bilirubin

A.

Explanation: The hepatic processing of bilirubin is accomplished in three key steps: 1) carriermediated uptake of bilirubin at the sinusoidal membrane: 2) conjugation of bilirubin with glucuronic acid by UGT (uridine diphosphate glucuronyl transferase) in the endoplasmic reticulum: and 3) biliary excretion of the water-soluble, nontoxic bilirubin glucuronides. Disruption of this process can be fatal, as seen with CriglerNajjar syndrome. Crigler-Najjar syndrome type 1 is an autosomal recessive disorder of bilirubin metabolism caused by a genetic lack of the LJGT enzyme needed to catalyze bile glucuronidation. When bilirubin is not correctly enzymatically processed by the liver, unconjugated hyperbilirubinemia develops. Indirect bilirubin levels typically approximate 20-25 mg/dL in these infants, but can rise to as high as 50 mg/dL. Conjugated bilirubin is water soluble, loosely bound to albumin, and excreted in urine when present in excess. In contrast, unconjugated bilirubin binds tightly to albumin and is highly insoluble in water. When bound, this unconjugated bilirubin cannot be filtered by the glomerulus and is therefore not excreted in the urine, Instead, the unconjugated bilirubin is gradually deposited into various tissues, including the brain. These deposits can cause kernicterus (bilirubin encephalopathy), which is a potentially fatal condition characterized by severe jaundice and neurologic impairment, as seen in this child. (Choice B) Dubin-Johnson syndrome is an autosomal recessive disorder characterized by the absence of a biliary transport protein, MRP2 (multidrug resistance protein 2), used in the hepatocellular excretion of bilirubin glucuronides into bile canaliculi. The liver is darkly pigmented as a result, but patients are typically asymptomatic and suffer no significant adverse effects. (Choice C) Rotor syndrome is a rare autosomal recessive disorder characterized by an asymptomatic conjugated hyperbilirubinemia that results from numerous defects in the hepatic uptake and excretion of bilirubin pigments. Although patients are often jaundiced, they enjoy normal life expectancies. (Choice D) High numbers of anaerobes and Staphylococcus aureus can result in the deconjugation (via removal of glycine and taurine) of bile acids, rendering them less soluble and therefore less able to form micelles. This deconjugation impedes the active reabsorption of bile acids into portal circulation at the terminal ileum, resulting in lipid malabsorption. However, there are no neurologic abnormalities associated with bilirubin deconjugation in the gut. (Choice E) Within the colon, bacterial enzymes reduce bilirubin into urobilinogen. A small fraction of these urobilinogen return to the liver through the enterohepatic circulation: these will later either re-enter the gastrointestinal tract or be excreted through the urine. The urobilinogen that remain in the colon are excreted in the feces as stercobilin, contributing to its dark color. Fortunately, there are no neurologic abnormalities associated with impaired gut reabsorption of bilirubin.

304

USMLE WORLD STEP 1

PATHOPHYSIOLOGY

Educational Objective: Crigler-Najjar syndrome is an autosomal recessive disorder of bilirubin metabolism caused by a genetic lack of the UGT enzyme needed to catalyze bile glucuronidation. Unconjugated hyperbilirubinemia develops in these infants, causing kernicterus and often death.

305

USMLE WORLD STEP 1

PATHOPHYSIOLOGY

Q NO 95: An 8-year-old boy with recurrent episodes of shortness of breath and wheezing is brought to the physician’s office for evaluation. Physical examination reveals mildly prolonged expirations. A complete blood count shows eosinophilia. The physician prescribes a pharmacologic receptor antagonist, and the symptoms improve substantially. The drug most likely acts at the receptor for: Serotonin B. Histamine C. Leukotriene D4 D. Platelet activating factor E. Prostaglandin E2

A.

Explanation: This patient’s history and physical exam findings are consistent with atopic (extrinsic allergic) asthma, the most common type of asthma. Atopic asthma usually manifests in childhood. Of the numerous inflammatory mediators postulated to play a role in the pathogenesis of allergic asthma, only leukotrienes (LTC4, LTC4, and LTC4) and acetylcholine produce bronchospasm that is relieved by pharmacologic antagonists. Leukotrienes are synthesized by mast cells, eosinophils, basophils, and other cell types that infiltrate the bronchial mucosa in asthmatics. In addition to causing bronchial constriction and hyperreactivity, leukotrienes also promote mucosal edema and mucus hypersecretion. Two leukotriene D4 receptor antagonists, zafirlukast and montelukast, may offer long-term control of atopic asthma by increasing airway caliber and reducing mucosal inflammation. Inhaled ipratropium is an antimuscarinic agent that blocks M3 receptors in airway smooth muscle and submucosal glands. It may offer short term relief of bronchoconstriction in allergic asthma. (Choice A) Serotonin does not play a major role in the pathogenesis of atopic asthma. (Choice B) Histamine is a bronchoconstrictor released by activated mast cells. Histamine may contribute to the pathogenesis of asthmatic bronchoconstriction, but its role appears to be relatively minor given that histamine antagonists lack efficacy in allergic asthma. (Choice D) RAE is a secondary inflammatory mediator released by activated mast cells that may promote bronchospasm and bronchial mucosal cellular infiltration. However, RAE receptor antagonists have not been shown tube as efficacious as leukotriene D4 and muscarinic receptor antagonists in the treatment of atopic asthma. (Choice E) Although prostaglandins may contribute to the pathogenesis of allergic asthma, prostaglandin receptor antagonists are less effective therapies than leukotriene D4 and muscarinic receptor antagonists in the treatment of atopic asthma. (Choice E) Although prostaglandins may contribute to the pathogenesis of allergic asthma, prostaglandin receptor antagonists are less effective therapies than leukotriene D4 and muscarinic receptor antagonists. Educational Objective: Although numerous substances are thought to play a role in the pathogenesis of allergic asthma, only leukotrienes (LTC4, LTD4, and LTE4) and acetylcholine have pharmacologic receptor antagonists that offer clear therapeutic benefit.

306

USMLE WORLD STEP 1

PATHOPHYSIOLOGY

Q NO 96: A 67-year-old Caucasian male presents to your office with right tibial pain that started three months ago and has increased in intensity overtime. His past medical history is significant for bilateral hearing impairment, diagnosed one year ago, and long-term hypertension. Physical examination reveals local tenderness and a lumpy protuberance over the right tibia. After extensive evaluation you proceed with bone biopsy. The pathologist calls to tell you that there were numerous multinucleate cells in the biopsy; some cells had over 100 nuclei that were positive for tartrate resistant acid phosphatase. Which of the following substances is essential for the differentiation of the cells described by the pathologist? A. Fibroblast growth factor (FGE) B. Transforming growth factor beta (TGF beta) C. Insulin-like growth factor (IGF) D. Macrophage colony-stimulating factor (M-CSF) E. Osteocalcin Explanation: This patient has clinical features suggestive of Paget’s disease of the bone: the patient is an older gentleman with pain and deformity in a bony area and hearing loss suggesting that the disease is affecting not only the patient’s right tibia, but also his skull. (There is also a Paget’s disease of the breast which is unrelated to the bone disease.) The primary abnormality in Paget’s disease of the bone is excessive osteoclastic bone resorption. A childhood infection of osteoclasts by a paramyxovirus might be responsible for Paget’s disease. Osteoclasts in Paget’s disease are typically very large and can have up to 100 nuclei (normal osteoclasts have 2-5 nuclei). The bone turnover is markedly increased in Paget’s disease, culminating in chaotic bone formation. Osteoclasts are the large multinucleated cells responsible for bone resorption. They originate from the mononuclear phagocytic cell lineage. Osteoclasts are formed when several precursor cells fuse, creating a multinucleated, mature cell. The osteoclastic plasma membrane that faces the calcified surface is called the ruffled border because it has deep folding. Lysosomal enzymes responsible for bone resorption are predominantly secreted through the ruffled border. Systemic hormones, as well as locally-acting cytokines, regulate the function of osteoclasts. The two most important factors for osteoclastic differentiation are produced by the osteoblast. These factors are M-CSF (monocyte colony-stimulating factor) and RANK-L (the receptor for activated nuclear factor kappa beta -ligand). Osteoclast precursors and mature osteoclasts posses receptors for RANK-L and M-CSF. (Choice A) Both acidic and basic fibroblast growth factors increase bone formation by stimulation of the osteoblast. Fibroblast growth factor leads to increased bone matrix formation. Fibroblast growth factor is also important for neovascularization and for wound healing. This neovascularization and matrix formation are also important in fracture repair. Abnormalities in the fibroblast growth factor receptor in the bone cause a form of congenital dwarfism known as “achondroplasia”. (Choice B) Transforming growth factor beta increases the replication of osteoblast precursors leading to increased formation of mature osteoblasts. Transforming growth factor beta also increases collagen synthesis and decreases bone resorption by increasing osteoclastic apoptosis. (Choice C) Insulin like growth factors (IGF-I and —II) are synthesized by various tissues including the liver and bone. IGF-l levels are much higher postnatally than IGF-II. IGF-I increases osteoblastic replication and collagen synthesis. IGF-I also

307

USMLE WORLD STEP 1

PATHOPHYSIOLOGY

decreases collagen degradation by inhibiting the enzyme matrix metalloproteinase-1 3 (MMP-13). The net effect of IGF-I on the bone is anabolic. (Choice E) Osteocalcin is a non-collagenous protein secreted by the osteoblast. It is used as a marker of bone formation. The exact function of osteocalcin is not known but it is thought to limit bone mineralization. In mice with deficient osteocalcin, the bone mass is increased. Educational Objective: Osteoclasts originate from hematopoietic progenitor cells. RANK-L and M-CSF play an important role in osteoclast differentiation. Paget’s disease of the bone is caused by excessive osteoclastic resorption.

308

USMLE WORLD STEP 1

PATHOPHYSIOLOGY

Q NO 97: A 43-year-old Caucasian male presents to your office with a severalmonth history of pruritus, rash pushing, and abdominal cramps. Small bowel biopsy demonstrates nests of mast cells within the mucosa. Which of the following additional findings would you expect most in this patient? Gastric atrophy B. Pernicious anemia C. Gastric hypersecretion D. Gastric hypomotility E. Bacterial colonization of the stomach F. Pancreatic tumor

A.

Explanation: Gastric acid is secreted by parietal cells located in the mucosal glands of the fundus and body of the stomach. It is stimulated by the following substances: 1. Histamine acts via H2 receptors and increases intracellular cAMP concentration. 2. Acetylcholine acts via M3 muscarinic receptors, and leads to an increase in intracellular calcium. 3. Gastrin binds to the gastrin receptor on the surface of parietal cells and increases intracellular calcium concentration. It also stimulates histamine synthesis and release by ECL (enterochromaffin-like) cells in the stomach. The latter is the most potent mechanism of gastrin action. Intracellular calcium and cAMP activate protein kinases and lead to increased transport of H+ by H+-K+ ATPase into the gastric lumen. In systemic mastocytosis, mast cell proliferation occurs in the bone marrow and in other organs. Increased histamine secretion ensues; as a result, many clinical symptoms of this disease are mediated by histamine. Gastric acid secretion increases, which inactivates pancreatic and intestinal enzymes, causing diarrhea. Other GI symptoms include nausea, vomiting, and abdominal cramps. Gastric ulcerations may occur. Other histamine-mediated symptoms of mastocytosis are syncope, flushing hypotension, tachycardia, and bronchospasm. Skin manifestations, such as pruritus, urticaria, and dermatographism, are typical. (Choice A) Gastric mucosal atrophy results from long-standing gastritis, most commonly associated with H. pylon. It usually clinically silent or manifests with nonspecific symptoms of nausea, vomiting, and epigastric discomfort. (Choice B) Pernicious anemia is associated with rare autoimmune gastritis. It results from autoantibody attack of parietal cells / intrinsic factor. If intrinsic factor is not produced, vitamin B 12 can not be absorbed, causing neurological symptoms. (Choice D) Gastric hypomotility (gastroparesis) occurs in patients with diabetes mellitus, uremia, hypothyroidism, and other metabolic disorders. Gastroparesis may cause constipation, episodic diarrhea, early satiety, and food stasis with subsequent vomiting. (Choice E) Colonization of the stomach with the gram-negative rod H. pylori is the most common cause of chronic gastritis and peptic ulcer disease. It is not associated with cutaneous pruritus and flushing. Mast cell proliferation is not associated with H pylon. (Choice F) Pancreatic tumors may secrete gastrin (Zollinger-Ellison syndrome), insulin (causing symptoms of fasting hypoglycemia), glucagon (causing diabetes mellitus and typical rash), and somatostatin (associated with diarrhea, cholelithiasis and diabetes mellitus). Pancreatic tumors, however, are not known to secrete histamine.

309

USMLE WORLD STEP 1

PATHOPHYSIOLOGY

Educational Objective: Systemic mastocytosis is characterized by the abnormal proliferation of mast cells and increased histamine secretion. Histamine increases the production of gastric acid by parietal cells. Gastric hypersecretion, therefore, is a common occurrence in systemic mastocytosis.

310

USMLE WORLD STEP 1

PATHOPHYSIOLOGY

Q NO 98: A 62-year-old male complains of progressive dyspnea that limits his daily activities. He has smoked 1-2 packs of cigarettes a day for over 30 years. Physical examination reveals bilaterally decreased breath sounds and scattered wheezes. His chest x-ray is shown below. Which of the following parameters is most likely to be increased in this patient?

A. Expiratory flow rates B. Carbon monoxide diffusion capacity C. Lung elastic recoil D. Forced vital capacity E. Functional residual capacity

Explanation: This patient’s clinical picture is consistent with chronic obstructive pulmonary disease (COPD). The chest x-ray shows hyperinflated lungs and a flattened diaphragm. COPD encompasses chronic bronchitis and/or emphysema. Patients with emphysema often have a barrel-shaped chest and decreased breath sounds on physical exam, as well as increases in both total lung capacity (TLC) and residual volume (RV). These spirometry findings are due to destruction of interalveolar walls and diminished lung elastic recoil which cause “air trapping” in enlarged distal airspaces. Since the functional residual capacity (RC) is equal to the expiratory reserve volume (ERV) plus the RV, and since the FRV is likely to be increased by the air trapping that characterizes emphysema, FRC will also tend to be increased in emphysematous patients. (Choice A) In both types of COPD expiratory airflow rates are reduced. This is because of the inflammatory and fibrotic narrowing of bronchi in chronic bronchitis and the decreased lung elastic recoil and increased dynamic airway compression during expiration in emphysema. (Choice B)The diffusing capacity of the lung for carbon monoxide (DLCO) depends on the thickness and total surface area of the alveolar capillary membrane and on the hematocrit and total volume of the pulmonary capillary blood. In emphysema, interalveolar wall destruction decreases the alveolar-capillary surface area, reducing diffusing capacity. In patients with emphysema, DLCO may be decreased even when there is little evidence of expiratory airflow obstruction by spirometric pulmonary function testing. (Choice C) In emphysema, lung elastic recoil is decreased due to destruction of interalveolar walls. (Choice D) In CQPDI forced vital capacity is generally decreased due to reduced lung elastic recoil and/or expiratory airflow obstruction.

311

USMLE WORLD STEP 1

PATHOPHYSIOLOGY

Educational Objective: In patients with emphysema, total lung capacity, residual volume, and functional residual capacity are generally increased, due to decreased lung elastic recoil. Air trapping in emphysematous patients tends to increase expiratory reserve volume and there by further contributes to an increased FRO.

312

USMLE WORLD STEP 1

PATHOPHYSIOLOGY

Q NO 99: A 62-year-old male visits his physician for evaluation of progressive exertional dyspnea. His past medical history is significant for pneumonia and hypertension. He has a 30 pack-year smoking history. On physical examination, breath sounds are decreased bilaterally, and wheezes are heard. A CT scan of his chest is shown below: Which of the following cell types was likely a major contributor to this patient’s lung condition?

A. Eosinophils B. Mast cells C. Neutrophils D. Type I pneumocytes E. Type II pneumocytes F. Clara cells

Explanation: A heavy smoker with exertional dyspnea and airspace enlargement on CT likely has centriacinar emphysema. The pathogenesis of centriacinar emphysema begins with oxidative injury to the respiratory bronchioles and activation of resident macrophages by components of cigarette smoke. Inflammatory recruitment of neutrophils into these airspaces follows. Neutrophils release neutrophil elastase, proteinase 3, cathepsin G, and matrix metalloproteinases. Activated macrophages also release proteases. Stimulated neutrophils generate oxygen free radicals as well, which inhibit the antiprotease activity of a1-antitrypsin. The resultant net proteaseantiprotease imbalance and oxidant-antioxidant imbalance destroys acinar walls. Excess neutrophil elastase activity in particular is a major factor in the development of centriacinar emphysema. (Neutrophil elastase is also important in the development of panacinar emphysema. Panacinar emphysema most commonly develops in individuals deficient in a1-antitrypsin, the major inhibitor of neutrophil elastase.) (Choice A) Eosinophils play an important role in the late phase component of Type I immediate hypersensitivity reactions. In the lung, they are recruited to help generate and sustain the localized inflammatory response that follows exposure to allergens. (Choice B) Interstitial pulmonary mast cells play a central role in the pathogenesis of allergic asthma, particularly via release of histamine and leukotrienes that induce bronchospasm. (Choice D) Type I pneumocytes constitute over 95% of the inner epithelial lining of the alveoli. This cell type is a target in the acinar wall destruction that occurs in emphysema. (Choice E) Type II pneumocytes are the source of pulmonary surfactant and the main cell type responsible for repair of alveolar epithelium after destruction of type I cells. (Choice F) Clara cells are non-ciliated, secretory constituents of the terminal respiratory epithelium. They secrete Clara cell secretory protein (CCSP) which

313

USMLE WORLD STEP 1

PATHOPHYSIOLOGY

inhibits neutrophil recruitment and activation as well as neutrophil dependent mucin production. Educational Objective: The pathogenesis of centriacinar emphysema associated with chronic, heavy smoking predominantly involves intraalveolar release of proteases, especially elastase, from infiltrating neutrophils and from alveolar macrophages.

314

USMLE WORLD STEP 1

PATHOPHYSIOLOGY

Q NO 100: A 72-year-old Caucasian male is hospitalized after complaining of chronic fatigue and exertional dyspnea. His left ventricular pressure-volume curve is pictured below (curve 2). Assuming heart failure is responsible for this patient’s symptoms which of the following is the most likely cause of this patient’s condition?

A. Viral myocarditis B. Alcoholic cardiomyopathy C. Amyloidosis D. Diphtheritic myocarditis E. High-dose doxorubicin

Explanation: This patient’s left ventricular (LV) diastolic pressure-volume curve (#2) shows reduced LV compliance (= dV/dP), indicating that diastolic dysfunction is contributing significantly to the patient’s congestive heart failure. Of the various etiologies for cardiomyopathy listed here, only amyloidosis can cause restrictive cardiomyopathy. In restrictive cardiomyopathy due to amyloidosis, there is infiltration of the myocardium with amyloid, making it stiffer. Diastolic dysfunction is the predominant mechanism of heart failure in restrictive cardiomyopathy. Restrictive cardiomyopathies may be idiopathic or due to amyloidosis, sarcoidosis, metastatic cancer, or products of inborn metabolic errors. (Choice A) A viral myocarditis may ultimately result in a dilated cardiomyopathy, with systolic dysfunction as the main mechanism of heart failure. (Choice B) Alcohol toxicity causes a dilated cardiomyopathy, with systolic dysfunction as the main mechanism of heart failure. (Choice D) Diphtheritic myocarditis may ultimately progress to a dilated cardiomyopathy, with systolic dysfunction as the main mechanism of heart failure. (Choice E) Certain chemotherapeutic agents, such as doxorubicin (Adriamycin) and daunorubicin, can cause dose- dependent cardiotoxicity that produces a dilated cardiomyopathy. Thus, systolic dysfunction would be the main mechanism of heart failure. Educational Objective: Heart failure due to left ventricular (LV) diastolic dysfunction is the result of a decrease in diastolic LV compliance. Restrictive cardiomyopathy, as can be caused by amyloidosis, may cause diastolic dysfunction. Infectious myocarditis and cardiotoxic agents including alcohol and doxorubicin tend to produce a dilated cardiomyopathy with predominantly systolic dysfunction.

315

View more...

Comments

Copyright ©2017 KUPDF Inc.
SUPPORT KUPDF